2004 AP English Literature and Composition Free …bayareatutoring.com/APlitprompts.pdf · AP®...

80
AP ® English Literature and Composition 2004 Free-Response Questions Form B The College Board is a not-for-profit membership association whose mission is to connect students to college success and opportunity. Founded in 1900, the association is composed of more than 4,500 schools, colleges, universities, and other educational organizations. Each year, the College Board serves over three million students and their parents, 23,000 high schools, and 3,500 colleges through major programs and services in college admissions, guidance, assessment, financial aid, enrollment, and teaching and learning. Among its best-known programs are the SAT ® , the PSAT/NMSQT ® , and the Advanced Placement Program ® (AP ® ). The College Board is committed to the principles of excellence and equity, and that commitment is embodied in all of its programs, services, activities, and concerns. For further information, visit www.collegeboard.com Copyright © 2004 College Entrance Examination Board. All rights reserved. College Board, Advanced Placement Program, AP, AP Central, AP Vertical Teams, APCD, Pacesetter, Pre-AP, SAT, Student Search Service, and the acorn logo are registered trademarks of the College Entrance Examination Board. PSAT/NMSQT is a registered trademark jointly owned by the College Entrance Examination Board and the National Merit Scholarship Corporation. Educational Testing Service and ETS are registered trademarks of Educational Testing Service. Other products and services may be trademarks of their respective owners. For the College Board’s online home for AP professionals, visit AP Central at apcentral.collegeboard.com. The materials included in these files are intended for noncommercial use by AP teachers for course and exam preparation; permission for any other use must be sought from the Advanced Placement Program ® . Teachers may reproduce them, in whole or in part, in limited quantities, for face-to-face teaching purposes but may not mass distribute the materials, electronically or otherwise. This permission does not apply to any third-party copyrights contained herein. These materials and any copies made of them may not be resold, and the copyright notices must be retained as they appear here.

Transcript of 2004 AP English Literature and Composition Free …bayareatutoring.com/APlitprompts.pdf · AP®...

Page 1: 2004 AP English Literature and Composition Free …bayareatutoring.com/APlitprompts.pdf · AP® English Literature and Composition 2004 Free-Response Questions Form B ... present,

AP® English Literature and Composition2004 Free-Response Questions

Form B

The College Board is a not-for-profit membership association whose mission is to connect students to college success and opportunity. Founded in 1900, the association is composed of more than 4,500 schools, colleges, universities, and other educational organizations. Each year, the College Board serves over three million students and their parents, 23,000 high schools, and 3,500 colleges through major programs and services in college admissions, guidance, assessment, financial aid, enrollment, and teaching and learning. Among its best-known programs are the SAT®, the

PSAT/NMSQT®, and the Advanced Placement Program® (AP®). The College Board is committed to the principles of excellence and equity, and that commitment is embodied in all of its programs, services, activities, and concerns.

For further information, visit www.collegeboard.com

Copyright © 2004 College Entrance Examination Board. All rights reserved. College Board, Advanced Placement Program, AP, AP Central,

AP Vertical Teams, APCD, Pacesetter, Pre-AP, SAT, Student Search Service, and the acorn logo are registered trademarks of the College Entrance Examination Board. PSAT/NMSQT is a registered trademark jointly owned by the

College Entrance Examination Board and the National Merit Scholarship Corporation. Educational Testing Service and ETS are registered trademarks of Educational Testing Service.

Other products and services may be trademarks of their respective owners.

For the College Board’s online home for AP professionals, visit AP Central at apcentral.collegeboard.com.

The materials included in these files are intended for noncommercial use by AP teachers for course and exam preparation; permission for any other use

must be sought from the Advanced Placement Program®. Teachers may reproduce them, in whole or in part, in limited quantities, for face-to-face

teaching purposes but may not mass distribute the materials, electronically or otherwise. This permission does not apply to any

third-party copyrights contained herein. These materials and any copies made of them may not be resold, and the copyright notices

must be retained as they appear here.

Page 2: 2004 AP English Literature and Composition Free …bayareatutoring.com/APlitprompts.pdf · AP® English Literature and Composition 2004 Free-Response Questions Form B ... present,

2004 AP® ENGLISH LITERATURE AND COMPOSITION FREE-RESPONSE QUESTIONS (Form B)

Copyright © 2004 by College Entrance Examination Board. All rights reserved. Visit apcentral.collegeboard.com (for AP professionals) and www.collegeboard.com/apstudents (for AP students and parents).

GO ON TO THE NEXT PAGE.

2

ENGLISH LITERATURE AND COMPOSITION SECTION II

Total time—2 hours Question 1

(Suggested time—40 minutes. This question counts as one-third of the total essay section score.)

The following passage comes from Elizabeth Gaskell’s Mary Barton (1848), a novel about mill workers living in Manchester, England, in the 1840’s. In this scene, George Wilson, one of the workers, goes to the house of Mr. Carson, the mill owner, to request care for a fellow worker dying of typhus. Read the passage carefully. Then, in a well-written essay, analyze how Gaskell uses elements such as point of view, selection of detail, dialogue, and characterization to make a social commentary. Wilson had about two miles to walk before he reached Mr Carson’s house, which was almost in the country. The streets were not yet bustling and busy. The shop-men were lazily taking down the shutters, although it was near eight o’clock; for the day was 5

long enough for the purchases people made in that quarter of the town, while trade was so flat. One or two miserable-looking women were setting off on their day’s begging expedition. But there were few people abroad. Mr Carson’s was a good house, and 10

furnished with disregard to expense. But in addition to lavish expenditure, there was much taste shown, and many articles chosen for their beauty and elegance adorned his rooms. As Wilson passed a window which a housemaid had thrown open, he saw pictures 15

and gilding, at which he was tempted to stop and look; but then he thought it would not be respectful. So he hastened on to the kitchen door. The servants seemed very busy with preparations for breakfast; but good-naturedly, though hastily, told him to step in, 20

and they could soon let Mr Carson know he was there. So he was ushered into a kitchen hung round with glittering tins, where a roaring fire burnt merrily, and where numbers of utensils hung round, at whose nature and use Wilson amused himself by guessing. 25

Meanwhile, the servants bustled to and fro; an out-door man-servant came in for orders, and sat down near Wilson; the cook broiled steaks, and the kitchen-maid toasted bread, and boiled eggs.

The coffee steamed upon the fire, and altogether 30

the odours were so mixed and appetizing, that Wilson began to yearn for food to break his fast, which had lasted since dinner1 the day before. If the servants had known this, they would have willingly given him meat and bread in abundance; but they were like the 35

rest of us, and not feeling hunger themselves, forgot it was possible another might. So Wilson’s craving turned to sickness, while they chattered on, making the kitchen’s free and keen remarks upon the parlour. ‘How late you were last night, Thomas!’ 40

‘Yes, I was right weary of waiting; they told me to be at the rooms by twelve; and there I was. But it was two o’clock before they called me.’ ‘And did you wait all that time in the street?’ asked the housemaid who had done her work for the 45

present, and come into the kitchen for a bit of gossip. ‘My eye as like! you don’t think I’m such a fool as to catch my death of cold, and let the horses catch their death too, as we should ha’ done if we’d stopped there. No! I put th’ horses up in th’ stables at th’ 50

Spread Eagle, and went mysel’, and got a glass or two by th’ fire. They’re driving a good custom, them, wi’ coachmen. There were five on us, and we’d many a quart o’ ale, and gin wi’ it, to keep out cold.’ ‘Mercy on us, Thomas; you’ll get a drunkard at 55

last!’

Line

Page 3: 2004 AP English Literature and Composition Free …bayareatutoring.com/APlitprompts.pdf · AP® English Literature and Composition 2004 Free-Response Questions Form B ... present,

2004 AP® ENGLISH LITERATURE AND COMPOSITION FREE-RESPONSE QUESTIONS (Form B)

Copyright © 2004 by College Entrance Examination Board. All rights reserved. Visit apcentral.collegeboard.com (for AP professionals) and www.collegeboard.com/apstudents (for AP students and parents).

GO ON TO THE NEXT PAGE.

3

‘If I do, I know whose blame it will be. It will be missis’s, and not mine. Flesh and blood can’t sit to be starved to death on a coach-box, waiting for folks as don’t know their own mind.’ 60

A servant, semi-upper-housemaid, semi-lady’s-maid, now came down with orders from her mistress. ‘Thomas, you must ride to the fishmonger’s, and say missis can’t give above half-a-crown a pound for salmon for Tuesday; she’s grumbling because trade’s 65

so bad. And she’ll want the carriage at three to go to the lecture, Thomas; at the Royal Execution,2 you know.’ ‘Ay, ay, I know.’ ‘And you’d better all of you mind your P’s and 70

Q’s, for she’s very black this morning. She’s got a bad headache.’

‘It’s a pity Miss Jenkins is not here to match her. Lord! how she and missis did quarrel which had got the worst headaches, it was that Miss Jenkins left for; 75

she would not give up having bad headaches, and missis could not abide any one to have ’em but herself.’ ‘Missis will have her breakfast up-stairs, cook, and the cold partridge as was left yesterday, and put 80

plenty of cream in her coffee, and she thinks there’s a roll left, and she would like it well buttered.’ So saying, the maid left the kitchen to be ready to attend to the young ladies’ bell when they chose to ring, after their late assembly the night before. 85 1 the noonday meal 2 a lecture hall

Page 4: 2004 AP English Literature and Composition Free …bayareatutoring.com/APlitprompts.pdf · AP® English Literature and Composition 2004 Free-Response Questions Form B ... present,

2004 AP® ENGLISH LITERATURE AND COMPOSITION FREE-RESPONSE QUESTIONS (Form B)

Copyright © 2004 by College Entrance Examination Board. All rights reserved. Visit apcentral.collegeboard.com (for AP professionals) and www.collegeboard.com/apstudents (for AP students and parents).

GO ON TO THE NEXT PAGE.

4

Question 2

(Suggested time — 40 minutes. This question counts as one-third of the total essay section score.)

Read the following poem carefully. Then, in a well-written essay, analyze the techniques the poet uses to develop the relationship between the speaker and the swamp.

Crossing the Swamp Here is the endless wet thick cosmos, the center of everything — the nugget of dense sap, branching 5

vines, the dark burred faintly belching bogs. Here is swamp, here is struggle, 10

closure — pathless, seamless, peerless mud. My bones knock together at the pale joints, trying 15

for foothold, fingerhold, mindhold over such slick crossings, deep hipholes, hummocks* that sink silently 20

into the black, slack earthsoup. I feel not wet so much as painted and glittered with the fat grassy 25

mires, the rich and succulent marrows of earth — a poor dry stick given one more chance by the whims 30

of swamp water — a bough that still, after all these years, could take root, sprout, branch out, bud — make of its life a breathing 35

palace of leaves. * low mounds of earth

From AMERICAN PRIMITIVE by Mary Oliver. Copyright © 1978, 1979, 1980, 1981, 1982, 1983 by Mary Oliver; First Appeared in ATLANTIC MONTHLY (1980). By permission of Little, Brown and Company, (Inc.).

Line

Page 5: 2004 AP English Literature and Composition Free …bayareatutoring.com/APlitprompts.pdf · AP® English Literature and Composition 2004 Free-Response Questions Form B ... present,

2004 AP® ENGLISH LITERATURE AND COMPOSITION FREE-RESPONSE QUESTIONS (Form B)

Copyright © 2004 by College Entrance Examination Board. All rights reserved. Visit apcentral.collegeboard.com (for AP professionals) and www.collegeboard.com/apstudents (for AP students and parents).

5

Question 3

(Suggested time— 40 minutes. This question counts as one-third of the total essay section score.) The most important themes in literature are sometimes developed in scenes in which a death or deaths take place. Choose a novel or play and write a well-organized essay in which you show how a specific death scene helps to illuminate the meaning of the work as a whole. Avoid mere plot summary. You may select a work from the list below or another novel or play of comparable literary merit.

All the King’s Men Anna Karenina As I Lay Dying The Awakening Billy Budd Bleak House Bless Me, Ultima Catch-22 Crime and Punishment The Crucible A Farewell to Arms Ghosts The Great Gatsby Heart of Darkness The House of Mirth Jude the Obscure

King Lear Madame Bovary The Mill on the Floss Moby-Dick Mrs. Dalloway Native Son One Hundred Years of Solitude Othello The Scarlet Letter Slaughterhouse-Five Song of Solomon The Stone Angel The Stranger A Tale of Two Cities Their Eyes Were Watching God

END OF EXAMINATION

Page 6: 2004 AP English Literature and Composition Free …bayareatutoring.com/APlitprompts.pdf · AP® English Literature and Composition 2004 Free-Response Questions Form B ... present,

AP® English Literature and Composition2004 Free-Response Questions

The College Board is a not-for-profit membership association whose mission is to connect students to college success and opportunity. Founded in 1900, the association is composed of more than 4,500 schools, colleges, universities, and other educational organizations. Each year, the College Board serves over three million students and their parents, 23,000 high schools, and 3,500 colleges through major programs and services in college admissions, guidance, assessment, financial aid, enrollment, and teaching and learning. Among its best-known programs are the SAT®, the

PSAT/NMSQT®, and the Advanced Placement Program® (AP®). The College Board is committed to the principles of excellence and equity, and that commitment is embodied in all of its programs, services, activities, and concerns.

For further information, visit www.collegeboard.com

Copyright © 2004 College Entrance Examination Board. All rights reserved. College Board, Advanced Placement Program, AP, AP Central,

AP Vertical Teams, APCD, Pacesetter, Pre-AP, SAT, Student Search Service, and the acorn logo are registered trademarks of the College Entrance Examination Board. PSAT/NMSQT is a registered trademark jointly owned by the

College Entrance Examination Board and the National Merit Scholarship Corporation. Educational Testing Service and ETS are registered trademarks of Educational Testing Service.

Other products and services may be trademarks of their respective owners.

For the College Board’s online home for AP professionals, visit AP Central at apcentral.collegeboard.com.

The materials included in these files are intended for noncommercial use by AP teachers for course and exam preparation; permission for any other use

must be sought from the Advanced Placement Program®. Teachers may reproduce them, in whole or in part, in limited quantities, for face-to-face

teaching purposes but may not mass distribute the materials, electronically or otherwise. This permission does not apply to any

third-party copyrights contained herein. These materials and any copies made of them may not be resold, and the copyright notices

must be retained as they appear here.

Page 7: 2004 AP English Literature and Composition Free …bayareatutoring.com/APlitprompts.pdf · AP® English Literature and Composition 2004 Free-Response Questions Form B ... present,

2004 AP® ENGLISH LITERATURE AND COMPOSITION FREE-RESPONSE QUESTIONS

Copyright © 2004 by College Entrance Examination Board. All rights reserved. Visit apcentral.collegeboard.com (for AP professionals) and www.collegeboard.com/apstudents (for AP students and parents).

GO ON TO THE NEXT PAGE.

2

ENGLISH LITERATURE AND COMPOSITION SECTION II

Total time—2 hours

Question 1

(Suggested time—40 minutes. This question counts as one-third of the total essay section score.)

The poems below are concerned with darkness and night. Read each poem carefully. Then, in a well-written essay, compare and contrast the poems, analyzing the significance of dark or night in each. In your essay, consider elements such as point of view, imagery, and structure.

Acquainted with the Night I have been one acquainted with the night. I have walked out in rain—and back in rain. I have outwalked the furthest city light. I have looked down the saddest city lane. I have passed by the watchman on his beat 5 And dropped my eyes, unwilling to explain. I have stood still and stopped the sound of feet When far away an interrupted cry Came over houses from another street, But not to call me back or say good-by; 10 And further still at an unearthly height, One luminary clock against the sky Proclaimed the time was neither wrong nor right. I have been one acquainted with the night. —Robert Frost “Acquainted with the Night” from THE POETRY OF ROBERT FROST edited by Edward Connery Lathem. Copyright 1928, © 1969 by Henry Holt and Co., copyright 1956 by Robert Frost. Reprinted by permission of Henry Holt and Company, LLC.

Line

Unfortunately, we have been denied permission to

reproduce Emily Dickinson’s poem, “We grow accustomed to the Dark,” on this Web site.

The poem was reprinted from

The Poems of Emily Dickinson, ed. Thomas H. Johnson (Cambridge: Harvard

University Press, 1951).

Page 8: 2004 AP English Literature and Composition Free …bayareatutoring.com/APlitprompts.pdf · AP® English Literature and Composition 2004 Free-Response Questions Form B ... present,

2004 AP® ENGLISH LITERATURE AND COMPOSITION FREE-RESPONSE QUESTIONS

Copyright © 2004 by College Entrance Examination Board. All rights reserved. Visit apcentral.collegeboard.com (for AP professionals) and www.collegeboard.com/apstudents (for AP students and parents).

GO ON TO THE NEXT PAGE.

3

Question 2

(Suggested time — 40 minutes. This question counts as one-third of the total essay section score.)

The following passage comes from the opening of “The Pupil” (1891), a story by Henry James. Read the passage carefully. Then write an essay in which you analyze the author’s depiction of the three characters and the relationships among them. Pay particular attention to tone and point of view.

The poor young man hesitated and procrastinated: it cost him such an effort to broach the subject of terms, to speak of money to a person who spoke only of feelings and, as it were, of the aristocracy. Yet he was unwilling to take leave, treating his engagement 5

as settled, without some more conventional glance in that direction than he could find an opening for in the manner of the large, affable lady who sat there drawing a pair of soiled gants de Suède* through a fat, jewelled hand and, at once pressing and gliding, 10

repeated over and over everything but the thing he would have liked to hear. He would have liked to hear the figure of his salary; but just as he was nervously about to sound that note the little boy came back— the little boy Mrs. Moreen had sent out of the room to 15

fetch her fan. He came back without the fan, only with the casual observation that he couldn’t find it. As he dropped this cynical confession he looked straight and hard at the candidate for the honour of taking his education in hand. This personage reflected, some-20

what grimly, that the first thing he should have to teach his little charge would be to appear to address himself to his mother when he spoke to her —especially not to make her such an improper answer as that. 25

When Mrs. Moreen bethought herself of this pretext for getting rid of their companion, Pemberton supposed it was precisely to approach the delicate subject of his remuneration. But it had been only to say some things about her son which it was better that 30

a boy of eleven shouldn’t catch. They were extravagantly to his advantage, save when she lowered her voice to sigh, tapping her left side familiarly: “And all over-clouded by this, you know — all at the mercy of a weakness — !” 35

Pemberton gathered that the weakness was in the region of the heart. He had known the poor child was not robust: this was the basis on which he had been invited to treat, through an English lady, an Oxford

acquaintance, then at Nice, who happened to know 40

both his needs and those of the amiable American family looking out for something really superior in the way of a resident tutor.

The young man’s impression of his prospective pupil, who had first come into the room, as if to see 45

for himself, as soon as Pemberton was admitted, was not quite the soft solicitation the visitor had taken for granted. Morgan Moreen was, somehow, sickly without being delicate, and that he looked intelligent (it is true Pemberton wouldn’t have enjoyed his being 50

stupid), only added to the suggestion that, as with his big mouth and big ears he really couldn’t be called pretty, he might be unpleasant. Pemberton was modest — he was even timid; and the chance that his small scholar might prove cleverer than himself had 55

quite figured, to his nervousness, among the dangers of an untried experiment. He reflected, however, that these were risks one had to run when one accepted a position, as it was called, in a private family; when as yet one’s University honours had, pecuniarily 60

speaking, remained barren. At any rate, when Mrs. Moreen got up as if to intimate that, since it was understood he would enter upon his duties within the week she would let him off now, he succeeded, in spite of the presence of the child, in squeezing out a 65

phrase about the rate of payment. It was not the fault of the conscious smile which seemed a reference to the lady’s expensive identity, if the allusion did not sound rather vulgar. This was exactly because she became still more gracious to reply: “Oh, I can assure 70

you that all that will be quite regular.” Pemberton only wondered, while he took up his

hat, what “all that” was to amount to—people had such different ideas. Mrs. Moreen’s words, however, seemed to commit the family to a pledge definite 75

enough to elicit from the child a strange little comment, in the shape of the mocking, foreign ejaculation, “Oh, là-là!”

* suede gloves

Line

Page 9: 2004 AP English Literature and Composition Free …bayareatutoring.com/APlitprompts.pdf · AP® English Literature and Composition 2004 Free-Response Questions Form B ... present,

2004 AP® ENGLISH LITERATURE AND COMPOSITION FREE-RESPONSE QUESTIONS

Copyright © 2004 by College Entrance Examination Board. All rights reserved. Visit apcentral.collegeboard.com (for AP professionals) and www.collegeboard.com/apstudents (for AP students and parents).

4

Question 3

(Suggested time— 40 minutes. This question counts as one-third of the total essay section score.) Critic Roland Barthes has said, “Literature is the question minus the answer.” Choose a novel or play and, considering Barthes’ observation, write an essay in which you analyze a central question the work raises and the extent to which it offers any answers. Explain how the author’s treatment of this question affects your understanding of the work as a whole. Avoid mere plot summary. You may select a work from the list below or another novel or play of comparable literary merit.

Alias Grace All the King’s Men Candide Crime and Punishment Death of a Salesman Doctor Faustus Don Quixote A Gesture Life Ghosts Great Expectations The Great Gatsby Gulliver’s Travels Heart of Darkness Invisible Man Joe Turner’s Come and Gone King Lear Major Barbara

Middlemarch Moby-Dick Obasan Oedipus Rex Orlando A Portrait of the Artist as a Young Man Rosencrantz and Guildenstern Are Dead The Scarlet Letter Sister Carrie The Sound and the Fury Sula The Sun Also Rises Their Eyes Were Watching God The Things They Carried The Turn of the Screw Who’s Afraid of Virginia Woolf

END OF EXAMINATION

Page 10: 2004 AP English Literature and Composition Free …bayareatutoring.com/APlitprompts.pdf · AP® English Literature and Composition 2004 Free-Response Questions Form B ... present,

AP® English Literature and Composition 2005 Free-Response Questions

Form B

The College Board: Connecting Students to College Success

The College Board is a not-for-profit membership association whose mission is to connect students to college success and opportunity. Founded in 1900, the association is composed of more than 4,700 schools, colleges, universities, and other educational organizations. Each year, the College Board serves over three and a half million students and their parents, 23,000 high schools, and 3,500 colleges through major programs and services in college admissions, guidance, assessment, financial aid, enrollment, and teaching and learning. Among its best-known programs are the SAT®, the PSAT/NMSQT®, and the Advanced Placement Program® (AP®). The College Board is committed to the principles of excellence and equity, and that commitment is embodied in all of its programs, services, activities, and concerns.

Copyright © 2005 by College Board. All rights reserved. College Board, AP Central, APCD, Advanced Placement Program, AP, AP Vertical Teams, Pre-AP, SAT, and the acorn logo are registered trademarks of the College Entrance Examination Board. Admitted Class Evaluation Service, CollegeEd, Connect to college success, MyRoad, SAT Professional Development, SAT Readiness Program, and Setting the Cornerstones are trademarks owned by the College Entrance Examination Board. PSAT/NMSQT is a registered trademark of the College Entrance Examination Board and National Merit Scholarship Corporation. Other products and services may be trademarks of their respective owners. Permission to use copyrighted College Board materials may be requested online at: http://www.collegeboard.com/inquiry/cbpermit.html. Visit the College Board on the Web: www.collegeboard.com. AP Central is the official online home for the AP Program and Pre-AP: apcentral.collegeboard.com.

Page 11: 2004 AP English Literature and Composition Free …bayareatutoring.com/APlitprompts.pdf · AP® English Literature and Composition 2004 Free-Response Questions Form B ... present,

2005 AP® ENGLISH LITERATURE FREE-RESPONSE QUESTIONS (Form B)

Copyright © 2005 by College Entrance Examination Board. All rights reserved. Visit apcentral.collegeboard.com (for AP professionals) and www.collegeboard.com/apstudents (for AP students and parents).

GO ON TO THE NEXT PAGE. 2

ENGLISH LITERATURE AND COMPOSITION SECTION II

Total time—2 hours

Question 1

(Suggested time— 40 minutes. This question counts as one-third of the total essay section score.)

Carefully read the two poems below. Then in a well-organized essay compare the speakers’ reflections on their early morning surroundings and analyze the techniques the poets use to communicate the speakers’ different states of mind.

Five A.M. Still dark, the early morning breathes a soft sound above the fire. Hooded lights on porches lead past lawns, a hedge; I pass the house of the couple who have the baby, the yard with the little 5 dog; my feet pad and grit on the pavement, flicker past streetlights; my arms alternate easily to my pace. Where are my troubles? There are people in every country who never turn into killers, saints have built 10 sanctuaries on islands and in valleys, conquerors have quit and gone home, for thousands of years farmers have worked their fields. My feet begin the uphill curve where a thicket spills with birds every spring. 15 The air doesn’t stir. Rain touches my face. “Five A.M.”Copyright 1991, 1998 by the Estate of William Stafford. Reprinted from THE WAY IT IS: NEW & SELECTED POEMS with the permission of Graywolf Press, Saint Paul, Minnesota.

Five Flights Up Still dark. The unknown bird sits on his usual branch. The little dog next door barks in his sleep inquiringly, just once. Perhaps in his sleep, too, the bird inquires 5 once or twice, quavering. Questions—if that is what they are— answered directly, simply, by day itself. Enormous morning, ponderous, meticulous; 10 gray light streaking each bare branch, each single twig, along one side, making another tree, of glassy veins . . . The bird still sits there. Now he seems to yawn. The little black dog runs in his yard. 15 His owner’s voice arises, stern, “You ought to be ashamed!” What has he done? He bounces cheerfully up and down; he rushes in circles in the fallen leaves. 20 Obviously, he has no sense of shame. He and the bird know everything is answered, all taken care of, no need to ask again. —Yesterday brought to today so lightly! 25 (A yesterday I find almost impossible to lift.) “Five Flights Up” from THE COMPLETE POEMS 1927-1979 by Elizabeth Bishop. Copyright © 1979, 1983 by Alice Helen Methfessel. Used by permission of Farrar, Straus and Giroux, LLC. All rights reserved.

Line Line

Page 12: 2004 AP English Literature and Composition Free …bayareatutoring.com/APlitprompts.pdf · AP® English Literature and Composition 2004 Free-Response Questions Form B ... present,

2005 AP® ENGLISH LITERATURE FREE-RESPONSE QUESTIONS (Form B)

Copyright © 2005 by College Entrance Examination Board. All rights reserved. Visit apcentral.collegeboard.com (for AP professionals) and www.collegeboard.com/apstudents (for AP students and parents).

GO ON TO THE NEXT PAGE. 3

Question 2

(Suggested time— 40 minutes. This question counts as one-third of the total essay section score.)

Read the passage below and write an essay discussing how the characterization in the passage reflects the narrator’s attitude toward McTeague. In your essay, consider such elements as diction, tone, detail, and syntax.

Then one day at San Francisco had come the news of his mother’s death; she had left him some money—not much, but enough to set him up in business; so he had cut loose from the charlatan and had opened his “Dental Parlors” on Polk Street, an “accommodation 5 street” of small shops in the residence quarter of the town. Here he had slowly collected a clientele of butcher boys, shop girls, drug clerks, and car conductors. He made but few acquaintances. Polk Street called him the “Doctor” and spoke of his 10 enormous strength. For McTeague was a young giant, carrying his huge shock of blond hair six feet three inches from the ground; moving his immense limbs, heavy with ropes of muscle, slowly, ponderously. His hands were enormous, red, and covered with a fell of 15 stiff yellow hair; they were hard as wooden mallets, strong as vises, the hands of the old-time car-boy. Often he dispensed with forceps and extracted a refractory tooth with his thumb and finger. His head was square-cut, angular; the jaw salient, like that of 20 the carnivora.

McTeague’s mind was as his body, heavy, slow to act, sluggish. Yet there was nothing vicious about the man. Altogether he suggested the draught horse, immensely strong, stupid, docile, obedient. 25

When he opened his “Dental Parlors,” he felt that his life was a success, that he could hope for nothing better. In spite of the name, there was but one room. It was a corner room on the second floor over the branch post-office, and faced the street. McTeague 30 made it do for a bedroom as well, sleeping on the big bed-lounge against the wall opposite the window.

There was a washstand behind the screen in the corner where he manufactured his moulds. In the round bay window were his operating chair, his dental engine, 35 and the movable rack on which he laid out his instru-ments. Three chairs, a bargain at the second-hand store, ranged themselves against the wall with military precision underneath a steel engraving of the court of Lorenzo de’ Medici, which he had bought 40 because there were a great many figures in it for the money. Over the bed-lounge hung a rifle manu-facturer’s advertisement calendar which he never used. The other ornaments were a small marble-topped centre table covered with back numbers of 45 “The American System of Dentistry,” a stone pug dog sitting before the little stove, and a thermometer. A stand of shelves occupied one corner, filled with the seven volumes of “Allen’s Practical Dentist.” On the top shelf McTeague kept his concertina and a bag of 50 bird seed for the canary. The whole place exhaled a mingled odor of bedding, creosote, and ether.

But for one thing, McTeague would have been perfectly contented. Just outside his window was his signboard—a modest affair—that read: “Doctor 55 McTeague. Dental Parlors. Gas Given”; but that was all. It was his ambition, his dream, to have projecting from that corner window a huge gilded tooth, a molar with enormous prongs, something gorgeous and attractive. He would have it some day, on that he was 60 resolved; but as yet such a thing was far beyond his means.

Line

Page 13: 2004 AP English Literature and Composition Free …bayareatutoring.com/APlitprompts.pdf · AP® English Literature and Composition 2004 Free-Response Questions Form B ... present,

2005 AP® ENGLISH LITERATURE FREE-RESPONSE QUESTIONS (Form B)

Copyright © 2005 by College Entrance Examination Board. All rights reserved. Visit apcentral.collegeboard.com (for AP professionals) and www.collegeboard.com/apstudents (for AP students and parents).

4

Question 3

(Suggested time—40 minutes. This question counts as one-third of the total essay section score.)

One of the strongest human drives seems to be a desire for power. Write an essay in which you discuss how a character in a novel or a drama struggles to free himself or herself from the power of others or seeks to gain power over others. Be sure to demonstrate in your essay how the author uses this power struggle to enhance the meaning of the work. You may choose one of the works listed below or another work of comparable quality that is appropriate to the question.

Adventures of Huckleberry Finn Antigone Beloved Ceremony Crime and Punishment Fences Great Expectations The Great Gatsby Hedda Gabler In the Time of the Butterflies Jane Eyre Julius Caesar

Macbeth Moby-Dick Native Speaker A Portrait of the Artist as a Young Man Pygmalion The Scarlet Letter Song of Solomon The Tempest Their Eyes Were Watching God Tracks Typical American Wide Sargasso Sea

END OF EXAMINATION

Page 14: 2004 AP English Literature and Composition Free …bayareatutoring.com/APlitprompts.pdf · AP® English Literature and Composition 2004 Free-Response Questions Form B ... present,

AP® English Literature and Composition 2005 Free-Response Questions

The College Board: Connecting Students to College Success

The College Board is a not-for-profit membership association whose mission is to connect students to college success and opportunity. Founded in 1900, the association is composed of more than 4,700 schools, colleges, universities, and other educational organizations. Each year, the College Board serves over three and a half million students and their parents, 23,000 high schools, and 3,500 colleges through major programs and services in college admissions, guidance, assessment, financial aid, enrollment, and teaching and learning. Among its best-known programs are the SAT®, the PSAT/NMSQT®, and the Advanced Placement Program® (AP®). The College Board is committed to the principles of excellence and equity, and that commitment is embodied in all of its programs, services, activities, and concerns.

Copyright © 2005 by College Board. All rights reserved. College Board, AP Central, APCD, Advanced Placement Program, AP, AP Vertical Teams, Pre-AP, SAT, and the acorn logo are registered trademarks of the College Entrance Examination Board. Admitted Class Evaluation Service, CollegeEd, Connect to college success, MyRoad, SAT Professional Development, SAT Readiness Program, and Setting the Cornerstones are trademarks owned by the College Entrance Examination Board. PSAT/NMSQT is a registered trademark of the College Entrance Examination Board and National Merit Scholarship Corporation. Other products and services may be trademarks of their respective owners. Permission to use copyrighted College Board materials may be requested online at: http://www.collegeboard.com/inquiry/cbpermit.html. Visit the College Board on the Web: www.collegeboard.com. AP Central is the official online home for the AP Program and Pre-AP: apcentral.collegeboard.com.

Page 15: 2004 AP English Literature and Composition Free …bayareatutoring.com/APlitprompts.pdf · AP® English Literature and Composition 2004 Free-Response Questions Form B ... present,

2005 AP® ENGLISH LITERATURE AND COMPOSITION FREE-RESPONSE QUESTIONS

Copyright © 2005 by College Entrance Examination Board. All rights reserved. Visit apcentral.collegeboard.com (for AP professionals) and www.collegeboard.com/apstudents (for AP students and parents).

GO ON TO THE NEXT PAGE. 2

ENGLISH LITERATURE AND COMPOSITION SECTION II

Total time—2 hours

Question 1

(Suggested time—40 minutes. This question counts as one-third of the total essay section score.)

The poems below, published in 1789 and 1794, were written by William Blake in response to the condition of chimney sweeps. Usually small children, sweeps were forced inside chimneys to clean their interiors. Read the two poems carefully. Then, in a well-written essay, compare and contrast the two poems, taking into consideration the poetic techniques Blake uses in each. The Chimney Sweeper When my mother died I was very young, And my father sold me while yet my tongue Could scarcely cry “ ’weep! ’weep! ’weep! ’weep!”* So your chimneys I sweep & in soot I sleep. There’s little Tom Dacre, who cried when his head 5

That curl’d like a lambs back, was shav’d, so I said, “Hush, Tom! never mind it, for when your head’s bare, You know that the soot cannot spoil your white hair.” And so he was quiet, & that very night, As Tom was a-sleeping he had such a sight! 10

That thousands of sweepers, Dick, Joe, Ned, & Jack, Were all of them lock’d up in coffins of black; And by came an Angel who had a bright key, And he open’d the coffins & set them all free; Then down a green plain, leaping, laughing they run, 15

And wash in a river and shine in the Sun; Then naked & white, all their bags left behind, They rise upon clouds, and sport in the wind. And the Angel told Tom, if he’d be a good boy, He’d have God for his father & never want joy. 20

And so Tom awoke; and we rose in the dark And got with our bags & our brushes to work. Tho’ the morning was cold, Tom was happy & warm; So if all do their duty, they need not fear harm. * The child’s lisping attempt at the chimney sweep’s street cry,

“Sweep! Sweep!”

William Blake, “The Chimney Sweeper,” The Complete Poetry and Prose of William Blake, ed. David V. Erdman (1789; 1794; Berkeley: University of California Press, 1965).

(1789)

The Chimney Sweeper A little black thing among the snow Crying “ ’weep, ’weep,” in notes of woe! “Where are thy father & mother? say?” “They are both gone up to the church to pray. “Because I was happy upon the heath, 5

And smil’d among the winter’s snow; They clothéd me in the clothes of death, And taught me to sing the notes of woe. “And because I am happy, & dance & sing, They think they have done me no injury, 10

And are gone to praise God & his Priest & King, Who make up a heaven of our misery.” William Blake, “The Chimney Sweeper,” The Complete Poetry and Prose of William Blake, ed. David V. Erdman (1789; 1794; Berkeley: University of California Press, 1965).

(1794)

Line Line

Page 16: 2004 AP English Literature and Composition Free …bayareatutoring.com/APlitprompts.pdf · AP® English Literature and Composition 2004 Free-Response Questions Form B ... present,

2005 AP® ENGLISH LITERATURE AND COMPOSITION FREE-RESPONSE QUESTIONS

Copyright © 2005 by College Entrance Examination Board. All rights reserved. Visit apcentral.collegeboard.com (for AP professionals) and www.collegeboard.com/apstudents (for AP students and parents).

GO ON TO THE NEXT PAGE. 3

Question 2

(Suggested time — 40 minutes. This question counts as one-third of the total essay section score.)

Printed below is the complete text of a short story written in 1946 by Katharine Brush. Read the story carefully. Then write an essay in which you show how the author uses literary devices to achieve her purpose.

.

Unfortunately, we have been denied permission to

reproduce “Birthday Party” by Katharine Brush on this Web site.

The short story was originally published in

The New Yorker.

Page 17: 2004 AP English Literature and Composition Free …bayareatutoring.com/APlitprompts.pdf · AP® English Literature and Composition 2004 Free-Response Questions Form B ... present,

2005 AP® ENGLISH LITERATURE AND COMPOSITION FREE-RESPONSE QUESTIONS

Copyright © 2005 by College Entrance Examination Board. All rights reserved. Visit apcentral.collegeboard.com (for AP professionals) and www.collegeboard.com/apstudents (for AP students and parents).

4

Question 3

(Suggested time—40 minutes. This question counts as one-third of the total essay section score.)

In Kate Chopin’s The Awakening (1899), protagonist Edna Pontellier is said to possess “that outward existence which conforms, the inward life which questions.” In a novel or play that you have studied, identify a character who conforms outwardly while questioning inwardly. Then write an essay in which you analyze how this tension between outward conformity and inward questioning contributes to the meaning of the work. Avoid mere plot summary. You may select a work from the list below or another appropriate novel or play of comparable literary merit.

Adventures of Huckleberry Finn The Age of Innocence The American As You Like It The Autobiography of an Ex-Colored Man Billy Budd Bless Me, Ultima Brave New World Catch-22 The Color Purple The Crucible Death of a Salesman A Doll’s House Ethan Frome A Gesture Life Go Tell It On the Mountain Invisible Man

King Lear Madame Bovary Middlemarch Mrs. Dalloway 1984 Obasan One Day in the Life of Ivan Denisovich Persuasion A Portrait of the Artist as a Young Man The Portrait of a Lady Rosencrantz and Guildenstern Are Dead The Scarlet Letter Surfacing The Sun Also Rises Their Eyes Were Watching God Typical American

END OF EXAM

Page 18: 2004 AP English Literature and Composition Free …bayareatutoring.com/APlitprompts.pdf · AP® English Literature and Composition 2004 Free-Response Questions Form B ... present,

AP® English Literature and Composition 2006 Free-Response Questions

Form B

The College Board: Connecting Students to College Success

The College Board is a not-for-profit membership association whose mission is to connect students to college success and opportunity. Founded in 1900, the association is composed of more than 5,000 schools, colleges, universities, and other educational organizations. Each year, the College Board serves seven million students and their parents, 23,000 high schools, and 3,500 colleges through major programs and services in college admissions, guidance, assessment, financial aid, enrollment, and teaching and learning. Among its best-known programs are the SAT®, the PSAT/NMSQT®, and the Advanced Placement Program® (AP®). The College Board is committed to the principles of excellence and equity, and that commitment is embodied in all of its programs, services, activities, and concerns.

© 2006 The College Board. All rights reserved. College Board, AP Central, APCD, Advanced Placement Program, AP, AP Vertical Teams, Pre-AP, SAT, and the acorn logo are registered trademarks of the College Board. Admitted Class Evaluation Service, CollegeEd, connect to college success, MyRoad, SAT Professional Development, SAT Readiness Program, and Setting the Cornerstones are trademarks owned by the College Board. PSAT/NMSQT is a registered trademark of the College Board and National Merit Scholarship Corporation. All other products and services may be trademarks of their respective owners. Permission to use copyrighted College Board materials may be requested online at: www.collegeboard.com/inquiry/cbpermit.html. Visit the College Board on the Web: www.collegeboard.com. AP Central is the official online home for the AP Program: apcentral.collegeboard.com.

Page 19: 2004 AP English Literature and Composition Free …bayareatutoring.com/APlitprompts.pdf · AP® English Literature and Composition 2004 Free-Response Questions Form B ... present,

2006 AP® ENGLISH LITERATURE AND COMPOSITION FREE-RESPONSE QUESTIONS (Form B)

© 2006 The College Board. All rights reserved. Visit apcentral.collegeboard.com (for AP professionals) and www.collegeboard.com/apstudents (for students and parents).

GO ON TO THE NEXT PAGE. 2

ENGLISH LITERATURE AND COMPOSITION SECTION II

Total time—2 hours Question 1

(Suggested time—40 minutes. This question counts as one-third of the total essay section score.)

Read the following poem carefully. Then write an essay discussing how the poet uses literary techniques to reveal the speaker’s attitudes toward nature and the artist’s task.

To Paint a Water Lily A green level of lily leaves Roofs the pond’s chamber and paves The flies’ furious arena: study These, the two minds of this lady. First observe the air’s dragonfly 5

That eats meat, that bullets by Or stands in space to take aim; Others as dangerous comb the hum Under the trees. There are battle-shouts And death-cries everywhere hereabouts 10

But inaudible, so the eyes praise To see the colours of these flies Rainbow their arcs, spark, or settle Cooling like beads of molten metal Through the spectrum. Think what worse 15

Is the pond-bed’s matter of course; Prehistoric bedragonned times Crawl that darkness with Latin names, Have evolved no improvements there, Jaws for heads, the set stare, 20

Ignorant of age as of hour— Now paint the long-necked lily-flower Which, deep in both worlds, can be still As a painting, trembling hardly at all Though the dragonfly alight, 25

Whatever horror nudge her root. — “To Paint a Water Lily” from Collected Poems by Ted Hughes (1930–1998), published by Faber and Faber Ltd [www.faber.co.uk]. Reprinted by permission of the publisher.

Line

Page 20: 2004 AP English Literature and Composition Free …bayareatutoring.com/APlitprompts.pdf · AP® English Literature and Composition 2004 Free-Response Questions Form B ... present,

2006 AP® ENGLISH LITERATURE AND COMPOSITION FREE-RESPONSE QUESTIONS (Form B)

© 2006 The College Board. All rights reserved. Visit apcentral.collegeboard.com (for AP professionals) and www.collegeboard.com/apstudents (for students and parents).

GO ON TO THE NEXT PAGE. 3

Question 2

(Suggested time—40 minutes. This question counts as one-third of the total essay section score.)

Read the passage below, which comes from a nineteenth-century novel. Then, in a well-developed essay, discuss how the narrator’s style reveals his attitudes toward the people he describes.

Gentlefolks in general have a very awkward rock ahead in life—the rock ahead of their own idleness. Their lives being, for the most part, passed in looking about them for something to do, it is curious to see—especially when their tastes are of what is called the 5

intellectual sort—how often they drift blindfold into some nasty pursuit. Nine times out of ten they take to torturing something, or to spoiling something—and they firmly believe they are improving their minds, when the plain truth is, they are only making a mess 10

in the house. I have seen them (ladies, I am sorry to say, as well as gentlemen) go out, day after day, for example, with empty pill-boxes, and catch newts, and beetles, and spiders, and frogs, and come home and stick pins through the miserable wretches, or cut them 15

up, without a pang of remorse, into little pieces. You see my young master, or my young mistress, poring over one of the spider’s insides with a magnifying-glass; . . . and when you wonder what this cruel nastiness means, you are told that it means a taste in 20

my young master or my young mistress for natural history. Sometimes, again, you see them occupied for hours together in spoiling a pretty flower with pointed instruments, out of a stupid curiosity to know what the flower is made of. Is its colour any prettier, or its 25

scent any sweeter, when you do know? But there! the

poor souls must get through the time, you see—they must get through the time. You dabbled in nasty mud, and made pies, when you were a child; and you dabble in nasty science, and dissect spiders, and spoil 30

flowers, when you grow up. In the one case and in the other, the secret of it is, that you have got nothing to think of in your poor empty head, and nothing to do with your poor idle hands. And so it ends in your spoiling canvas with paints, and making a smell in 35

the house; or in keeping tadpoles in a glass box full of dirty water, and turning everybody’s stomach in the house; or in chipping off bits of stone here, there, and everywhere, and dropping grit into all the victuals in the house; or in staining your fingers in the pursuit 40

of photography, and doing justice without mercy on everybody’s face in the house. It often falls heavy enough, no doubt, on people who are really obliged to get their living, to be forced to work for the clothes that cover them, the roof that shelters them, and the 45

food that keeps them going. But compare the hardest day’s work you ever did with the idleness that splits flowers and pokes its way into spiders’ stomachs, and thank your stars that your head has got something it must think of, and your hands something that they 50

must do.

Line

Page 21: 2004 AP English Literature and Composition Free …bayareatutoring.com/APlitprompts.pdf · AP® English Literature and Composition 2004 Free-Response Questions Form B ... present,

2006 AP® ENGLISH LITERATURE AND COMPOSITION FREE-RESPONSE QUESTIONS (Form B)

© 2006 The College Board. All rights reserved. Visit apcentral.collegeboard.com (for AP professionals) and www.collegeboard.com/apstudents (for students and parents).

4

Question 3

(Suggested time—40 minutes. This question counts as one-third of the total essay section score.)

In many works of literature, a physical journey—the literal movement from one place to another—plays a central role. Choose a novel, play, or epic poem in which a physical journey is an important element and discuss how the journey adds to the meaning of the work as a whole. You may write your essay on one of the following works or on another of comparable quality. Avoid mere plot summary.

Adventures of Huckleberry Finn The Aeneid All the Pretty Horses As I Lay Dying Candide The Canterbury Tales Cold Mountain The Divine Comedy Dutchman Going After Cacciato Gulliver’s Travels

Heart of Darkness The Importance of Being Earnest Light in August Middle Passage Moby-Dick Mother Courage Obasan The Odyssey Peer Gynt Rosencrantz and Guildenstern Are Dead Song of Solomon

STOP

END OF EXAM

Page 22: 2004 AP English Literature and Composition Free …bayareatutoring.com/APlitprompts.pdf · AP® English Literature and Composition 2004 Free-Response Questions Form B ... present,

AP® English Literature and Composition 2006 Free-Response Questions

The College Board: Connecting Students to College Success

The College Board is a not-for-profit membership association whose mission is to connect students to college success and opportunity. Founded in 1900, the association is composed of more than 5,000 schools, colleges, universities, and other educational organizations. Each year, the College Board serves seven million students and their parents, 23,000 high schools, and 3,500 colleges through major programs and services in college admissions, guidance, assessment, financial aid, enrollment, and teaching and learning. Among its best-known programs are the SAT®, the PSAT/NMSQT®, and the Advanced Placement Program® (AP®). The College Board is committed to the principles of excellence and equity, and that commitment is embodied in all of its programs, services, activities, and concerns.

© 2006 The College Board. All rights reserved. College Board, AP Central, APCD, Advanced Placement Program, AP, AP Vertical Teams, Pre-AP, SAT, and the acorn logo are registered trademarks of the College Board. Admitted Class Evaluation Service, CollegeEd, connect to college success, MyRoad, SAT Professional Development, SAT Readiness Program, and Setting the Cornerstones are trademarks owned by the College Board. PSAT/NMSQT is a registered trademark of the College Board and National Merit Scholarship Corporation. All other products and services may be trademarks of their respective owners. Permission to use copyrighted College Board materials may be requested online at: www.collegeboard.com/inquiry/cbpermit.html. Visit the College Board on the Web: www.collegeboard.com. AP Central is the official online home for the AP Program: apcentral.collegeboard.com.

Page 23: 2004 AP English Literature and Composition Free …bayareatutoring.com/APlitprompts.pdf · AP® English Literature and Composition 2004 Free-Response Questions Form B ... present,

2006 AP® ENGLISH LITERATURE AND COMPOSITION FREE-RESPONSE QUESTIONS

© 2006 The College Board. All rights reserved. Visit apcentral.collegeboard.com (for AP professionals) and www.collegeboard.com/apstudents (for students and parents).

GO ON TO THE NEXT PAGE. 2

ENGLISH LITERATURE AND COMPOSITION

SECTION II Total time—2 hours

Question 1

(Suggested time—40 minutes. This question counts as one-third of the total essay section score.)

Read the following poem carefully. Then write a well-organized essay in which you analyze how the poet uses language to describe the scene and to convey mood and meaning.

Evening Hawk From plane of light to plane, wings dipping through Geometries and orchids that the sunset builds, Out of the peak’s black angularity of shadow, riding The last tumultuous avalanche of Light above pines and the guttural gorge, 5

The hawk comes. His wing Scythes down another day, his motion Is that of the honed steel-edge, we hear The crashless fall of stalks of Time. The head of each stalk is heavy with the gold of our error. 10

Look! look! he is climbing the last light Who knows neither Time nor error, and under Whose eye, unforgiving, the world, unforgiven, swings Into shadow. Long now, The last thrush is still, the last bat 15

Now cruises in his sharp hieroglyphics. His wisdom Is ancient, too, and immense. The star Is steady, like Plato,* over the mountain. If there were no wind we might, we think, hear The earth grind on its axis, or history 20

Drip in darkness like a leaking pipe in the cellar. —Robert Penn Warren

*Greek philosopher (427?–347? B.C.) Copyright © 1998 by Estate of Robert Penn Warren. Reprinted by permission of William Morris Agency, Inc. on behalf of Author.

Line

Page 24: 2004 AP English Literature and Composition Free …bayareatutoring.com/APlitprompts.pdf · AP® English Literature and Composition 2004 Free-Response Questions Form B ... present,

2006 AP® ENGLISH LITERATURE AND COMPOSITION FREE-RESPONSE QUESTIONS

© 2006 The College Board. All rights reserved. Visit apcentral.collegeboard.com (for AP professionals) and www.collegeboard.com/apstudents (for students and parents).

GO ON TO THE NEXT PAGE. 3

Question 2

(Suggested time—40 minutes. This question counts as one-third of the total essay section score.) The following passage is an excerpt from Lady Windermere’s Fan, a play by Oscar Wilde, produced in 1892. Read the passage carefully. Then write a well-organized essay in which you analyze how the playwright reveals the values of the characters and the nature of their society.

DUCHESS OF BERWICK (shaking hands). Dear Margaret, I am so pleased to see you. You remember Agatha,1 don’t you? How do you do, Lord Darlington? I won’t let you know my daughter, you are far too wicked. 5

LORD DARLINGTON. Don’t say that, Duchess. As a wicked man I am a complete failure. Why, there are lots of people who say I have never really done anything wrong in the whole course of my life. Of course they only say it behind my back. 10

DUCHESS OF BERWICK. Isn’t he dreadful? Agatha, this is Lord Darlington. Mind you don’t believe a word he says. No, no tea, thank you, dear. (Sits on sofa.) We have just had tea at Lady Markby’s. Such bad tea, too. It was quite undrinkable. I wasn’t at 15

all surprised. Her own son-in-law supplies it. Agatha is looking forward so much to your ball tonight, dear Margaret.

LADY WINDERMERE (seated). Oh, you musn’t think it is going to be a ball, Duchess. It is only a 20

dance in honour of my birthday. A small and early. LORD DARLINGTON (standing). Very small,

very early, and very select, Duchess. DUCHESS OF BERWICK. Of course it’s going

to be select. But we know that, dear Margaret, about 25

your house. It is really one of the few houses in London where I can take Agatha, and where I feel perfectly secure about dear Berwick. I don’t know what society is coming to. The most dreadful people seem to go everywhere. They certainly come to my 30

parties—the men get quite furious if one doesn’t ask them. Really, some one should make a stand against it.

LADY WINDERMERE. I will, Duchess. I will have no one in my house about whom there is any 35

scandal. LORD DARLINGTON. Oh, don’t say that, Lady

Windermere. I should never be admitted. (Sitting.) 1 the Duchess’s daughter

DUCHESS OF BERWICK. Oh, men don’t matter. With women it is different. We’re good. Some of us 40

are, at least. But we are positively getting elbowed into the corner. Our husbands would really forget our existence if we didn’t nag at them from time to time, just to remind them that we have a perfect legal right to do so. 45

LORD DARLINGTON. It’s a curious thing, Duchess, about the game of marriage—a game, by the way, that is going out of fashion—the wives hold all the honours2 and invariably lose the odd trick.3

DUCHESS OF BERWICK. The odd trick? Is that 50

the husband, Lord Darlington? LORD DARLINGTON. It would be rather a good

name for the modern husband. DUCHESS OF BERWICK. Dear Lord Darlington,

how thoroughly depraved you are! 55

LADY WINDERMERE. Lord Darlington is trivial. LORD DARLINGTON. Ah, don’t say that, Lady

Windermere. LADY WINDERMERE. Why do you talk so

trivially about life, then? 60

LORD DARLINGTON. Because I think that life is far too important a thing ever to talk seriously about it.

DUCHESS OF BERWICK. What does he mean? Do, as a concession to my poor wits, Lord Darlington, 65

just explain to me what you really mean. LORD DARLINGTON. I think I had better not,

Duchess. Nowadays to be intelligible is to be found out. Good-bye! (Shakes hands with DUCHESS.) And now—Lady Windermere, good-bye. I may come 70

tonight, mayn’t I? Do let me come. LADY WINDERMERE. Yes, certainly. But you

are not to say foolish, insincere things to people. LORD DARLINGTON (smiling). Ah! you are

beginning to reform me. It is a dangerous thing to 75

reform any one, Lady Windermere. (Bows and exit). 2 high cards 3 round of a card game

Line

Page 25: 2004 AP English Literature and Composition Free …bayareatutoring.com/APlitprompts.pdf · AP® English Literature and Composition 2004 Free-Response Questions Form B ... present,

2006 AP® ENGLISH LITERATURE AND COMPOSITION FREE-RESPONSE QUESTIONS

© 2006 The College Board. All rights reserved. Visit apcentral.collegeboard.com (for AP professionals) and www.collegeboard.com/apstudents (for students and parents).

4

Question 3

(Suggested time—40 minutes. This question counts as one-third of the total essay section score.) Many writers use a country setting to establish values within a work of literature. For example, the country may be a place of virtue and peace or one of primitivism and ignorance. Choose a novel or play in which such a setting plays a significant role. Then write an essay in which you analyze how the country setting functions in the work as a whole. Do not merely summarize the plot. You may choose a work from the list below or another appropriate novel or play of similar literary merit.

Adam Bede Adventures of Huckleberry Finn All the Pretty Horses Anna Karenina As I Lay Dying As You Like It The Bear Black Boy Bless Me, Ultima The Bonesetter’s Daughter Ceremony The Cherry Orchard David Copperfield Don Quixote East of Eden Ethan Frome For Whom the Bell Tolls Frankenstein

The Grapes of Wrath House Made of Dawn King Lear Madame Bovary Mansfield Park A Midsummer Night’s Dream Obasan O Pioneers! Out of Africa The Scarlet Letter Tess of the D’Urbervilles Their Eyes Were Watching God A Thousand Acres Tom Jones The Vicar of Wakefield The Way We Live Now The Winter’s Tale Wuthering Heights

STOP

END OF EXAM

Page 26: 2004 AP English Literature and Composition Free …bayareatutoring.com/APlitprompts.pdf · AP® English Literature and Composition 2004 Free-Response Questions Form B ... present,

AP® English Literature and Composition 2007 Free-Response Questions

Form B

The College Board: Connecting Students to College Success

The College Board is a not-for-profit membership association whose mission is to connect students to college success and opportunity. Founded in 1900, the association is composed of more than 5,000 schools, colleges, universities, and other educational organizations. Each year, the College Board serves seven million students and their parents, 23,000 high schools, and 3,500 colleges through major programs and services in college admissions, guidance, assessment, financial aid, enrollment, and teaching and learning. Among its best-known programs are the SAT®, the PSAT/NMSQT®, and the Advanced Placement Program® (AP®). The College Board is committed to the principles of excellence and equity, and that commitment is embodied in all of its programs, services, activities, and concerns.

© 2007 The College Board. All rights reserved. College Board, Advanced Placement Program, AP, AP Central, SAT, and the acorn logo are registered trademarks of the College Board. PSAT/NMSQT is a registered trademark of the College Board and National Merit Scholarship Corporation. Permission to use copyrighted College Board materials may be requested online at: www.collegeboard.com/inquiry/cbpermit.html. Visit the College Board on the Web: www.collegeboard.com. AP Central is the official online home for the AP Program: apcentral.collegeboard.com.

Page 27: 2004 AP English Literature and Composition Free …bayareatutoring.com/APlitprompts.pdf · AP® English Literature and Composition 2004 Free-Response Questions Form B ... present,

2007 AP® ENGLISH LITERATURE FREE-RESPONSE QUESTIONS (Form B)

© 2007 The College Board. All rights reserved. Visit apcentral.collegeboard.com (for AP professionals) and www.collegeboard.com/apstudents (for students and parents).

GO ON TO THE NEXT PAGE. -2-

ENGLISH LITERATURE AND COMPOSITION SECTION II

Total time—2 hours

Question 1

(Suggested time — 40 minutes. This question counts as one-third of the total essay section score.) Read the following poem carefully. Then, write a well-organized essay in which you analyze the techniques the

poet uses to convey his attitude toward the places he describes.

Here Swerving east, from rich industrial shadows And traffic all night north; swerving through fields Too thin and thistled to be called meadows, And now and then a harsh-named halt, that shields Workmen at dawn; swerving to solitude 5

Of skies and scarecrows, haystacks, hares and pheasants, And the widening river’s slow presence, The piled gold clouds, the shining gull-marked mud, Gathers to the surprise of a large town: Here domes and statues, spires and cranes cluster 10

Beside grain-scattered streets, barge-crowded water, And residents from raw estates, brought down The dead straight miles by stealing flat-faced trolleys, Push through plate-glass swing doors to their desires— Cheap suits, red kitchen-ware, sharp shoes, iced lollies, 15

Electric mixers, toasters, washers, driers— A cut-price crowd, urban yet simple, dwelling Where only salesmen and relations come Within a terminate and fishy-smelling Pastoral of ships up streets, the slave museum, 20

Tattoo-shops, consulates, grim head-scarfed wives; And out beyond its mortgaged half-built edges Fast-shadowed wheat-fields, running high as hedges, Isolate villages, where removed lives

Line

Page 28: 2004 AP English Literature and Composition Free …bayareatutoring.com/APlitprompts.pdf · AP® English Literature and Composition 2004 Free-Response Questions Form B ... present,

2007 AP® ENGLISH LITERATURE FREE-RESPONSE QUESTIONS (Form B)

© 2007 The College Board. All rights reserved. Visit apcentral.collegeboard.com (for AP professionals) and www.collegeboard.com/apstudents (for students and parents).

GO ON TO THE NEXT PAGE. -3-

Loneliness clarifies. Here silence stands 25

Like heat. Here leaves unnoticed thicken, Hidden weeds flower, neglected waters quicken, Luminously-peopled air ascends; And past the poppies bluish neutral distance Ends the land suddenly beyond a beach 30

Of shapes and shingle. Here is unfenced existence: Facing the sun, untalkative, out of reach.

(1964) “Here” from COLLECTED POEMS by Philip Larkin. Copyright © 1988, 1989 by the Estate of Philip Larkin. Reprinted by permission of The Society of Authors as the Literary Representative of the Estate of Philip Larkin.

Page 29: 2004 AP English Literature and Composition Free …bayareatutoring.com/APlitprompts.pdf · AP® English Literature and Composition 2004 Free-Response Questions Form B ... present,

2007 AP® ENGLISH LITERATURE FREE-RESPONSE QUESTIONS (Form B)

© 2007 The College Board. All rights reserved. Visit apcentral.collegeboard.com (for AP professionals) and www.collegeboard.com/apstudents (for students and parents).

GO ON TO THE NEXT PAGE. -4-

Question 2

(Suggested time—40 minutes. This question counts as one-third of the total essay section score.) In the following passage, contemporary novelist Seamus Deane reflects on his childhood experiences with books

and writing. Read the passage carefully. Then, in a well-written essay, analyze how Deane conveys the impact those early experiences had on him.

The novel was called The Shan Van Vocht, a

phonetic rendering of an Irish phrase meaning The Poor Old Woman, a traditional name for Ireland. It was about the great rebellion of 1798, the source of almost half the songs we sang around the August 5

bonfires on the Feast of the Assumption. In the opening pages, people were talking in whispers about the dangers of the rebellion as they sat around a great open-hearth fire on a wild night of winter rain and squall. I read and re-read the opening many times. 10

Outside was the bad weather; inside was the fire, implied danger, a love relationship. There was something exquisite in this blend, as I lay in bed reading while my brothers slept and shifted under the light that shone on their eyelids and made their 15

dreams different. The heroine was called Ann, and the hero was Robert. She was too good for him. When they whispered, she did all the interesting talking. He just kept on about dying and remembering her always, even when she was there in front of him with her dark 20

hair and her deep golden-brown eyes and her olive skin. So I talked to her instead and told her how beautiful she was and how I wouldn’t go out on the rebellion at all but just sit there and whisper in her ear and let her know that now was forever and not some 25

time in the future when the shooting and the hacking would be over, when what was left of life would be spent listening to the night wind wailing on grave-yards and empty hillsides.

“For Christ’s sake, put off that light. You’re not 30

even reading, you blank gom.” And Liam would turn over, driving his knees up

into my back and muttering curses under his breath. I’d switch off the light, get back in bed, and lie there, the book still open, re-imagining all I had read, the 35

various ways the plot might unravel, the novel opening into endless possibilities in the dark.

The English teacher read out a model essay which had been, to our surprise, written by a country boy. It was an account of his mother setting the table for the 40

evening meal and then waiting with him until his father came in from the fields. She put out a blue-and-white jug full of milk and a covered dish of potatoes in their jackets and a red-rimmed butter dish with a slab of butter, the shape of a swan dipping its head 45

imprinted on its surface. That was the meal. Every-thing was so simple, especially the way they waited. She sat with her hands in her lap and talked to him about someone up the road who had had an airmail letter from America. She told him that his father 50

would be tired, but, tired as he was, he wouldn’t be without a smile before he washed himself and he wouldn’t be so without his manners to forget to say grace before they ate and that he, the boy, should watch the way the father would smile when the 55

books were produced for homework, for learning was a wonder to him, especially the Latin. Then there would be no talking, just the ticking of the clock and the kettle humming and the china dogs on the mantelpiece looking, as ever, across at one another. 60

“Now that,” said the master, “that’s writing. That’s just telling the truth.”

I felt embarrassed because my own essay had been full of long or strange words I had found in the dictionary—“cerulean,” “azure,” “phantasm” and 65

“implacable”—all of them describing skies and seas I had seen only with the Ann of the novel. I’d never thought such stuff was worth writing about. It was ordinary life—no rebellions or love affairs or dangerous flights across the hills at night. And yet I 70

kept remembering that mother and son waiting in the Dutch interior of that essay, with the jug of milk and the butter on the table, while behind and above them were those wispy, shawly figures from the rebellion, sibilant above the great fire and below the aching, 75

high wind. (1996)

Line

Page 30: 2004 AP English Literature and Composition Free …bayareatutoring.com/APlitprompts.pdf · AP® English Literature and Composition 2004 Free-Response Questions Form B ... present,

2007 AP® ENGLISH LITERATURE FREE-RESPONSE QUESTIONS (Form B)

© 2007 The College Board. All rights reserved. Visit apcentral.collegeboard.com (for AP professionals) and www.collegeboard.com/apstudents (for students and parents).

-5-

Question 3

(Suggested time — 40 minutes. This question counts as one-third of the total essay section score.) Works of literature often depict acts of betrayal. Friends and even family may betray a protagonist; main characters may likewise be guilty of treachery or may betray their own values. Select a novel or play that includes such acts of betrayal. Then, in a well-written essay, analyze the nature of the betrayal and show how it contributes to the meaning of the work as a whole. Choose a work from the list below or another novel or play of comparable quality. Avoid mere plot summary. Adventures of Huckleberry Finn All the Pretty Horses As I Lay Dying The Awakening Billy Budd Catch-22 Ceremony Death of a Salesman Great Expectations The Great Gatsby Invisible Man Jane Eyre Julius Caesar Major Barbara The Mayor of Casterbridge

Murder in the Cathedral Othello The Portrait of a Lady Push A Raisin in the Sun The Return of the Native Song of Solomon The Sound and the Fury The Street Sula We Were the Mulvaneys Who’s Afraid of Virginia Woolf ? Wide Sargasso Sea Wuthering Heights

STOP

END OF EXAM

Page 31: 2004 AP English Literature and Composition Free …bayareatutoring.com/APlitprompts.pdf · AP® English Literature and Composition 2004 Free-Response Questions Form B ... present,

AP® English Literature and Composition 2007 Free-Response Questions

The College Board: Connecting Students to College Success

The College Board is a not-for-profit membership association whose mission is to connect students to college success and opportunity. Founded in 1900, the association is composed of more than 5,000 schools, colleges, universities, and other educational organizations. Each year, the College Board serves seven million students and their parents, 23,000 high schools, and 3,500 colleges through major programs and services in college admissions, guidance, assessment, financial aid, enrollment, and teaching and learning. Among its best-known programs are the SAT®, the PSAT/NMSQT®, and the Advanced Placement Program® (AP®). The College Board is committed to the principles of excellence and equity, and that commitment is embodied in all of its programs, services, activities, and concerns.

© 2007 The College Board. All rights reserved. College Board, Advanced Placement Program, AP, AP Central, SAT, and the acorn logo are registered trademarks of the College Board. PSAT/NMSQT is a registered trademark of the College Board and National Merit Scholarship Corporation. Permission to use copyrighted College Board materials may be requested online at: www.collegeboard.com/inquiry/cbpermit.html. Visit the College Board on the Web: www.collegeboard.com. AP Central is the official online home for the AP Program: apcentral.collegeboard.com.

Page 32: 2004 AP English Literature and Composition Free …bayareatutoring.com/APlitprompts.pdf · AP® English Literature and Composition 2004 Free-Response Questions Form B ... present,

2007 AP® ENGLISH LITERATURE AND COMPOSITION FREE-RESPONSE QUESTIONS

© 2007 The College Board. All rights reserved. Visit apcentral.collegeboard.com (for AP professionals) and www.collegeboard.com/apstudents (for students and parents).

GO ON TO THE NEXT PAGE. -2-

ENGLISH LITERATURE AND COMPOSITION SECTION II

Total time—2 hours

Question 1

(Suggested time—40 minutes. This question counts as one-third of the total essay section score.)

In the following two poems, adults provide explanations for children. Read the poems carefully. Then write an essay in which you compare and contrast the two poems, analyzing how each poet uses literary devices to make his point. A Barred Owl The warping night air having brought the boom Of an owl’s voice into her darkened room, We tell the wakened child that all she heard Was an odd question from a forest bird, Asking of us, if rightly listened to, 5

“Who cooks for you?” and then “Who cooks for you?” Words, which can make our terrors bravely clear, Can also thus domesticate a fear, And send a small child back to sleep at night Not listening for the sound of stealthy flight 10

Or dreaming of some small thing in a claw Borne up to some dark branch and eaten raw. —Richard Wilbur "A Barred Owl" from MAYFLIES: NEW POEMS AND TRANSLATIONS, copyright © 2000 by Richard Wilbur, reprinted by permission of Harcourt, Inc.

The History Teacher Trying to protect his students’ innocence he told them the Ice Age was really just the Chilly Age, a period of a million years when everyone had to wear sweaters. And the Stone Age became the Gravel Age, 5

named after the long driveways of the time. The Spanish Inquisition was nothing more than an outbreak of questions such as “How far is it from here to Madrid?” “What do you call the matador’s hat?” 10

The War of the Roses took place in a garden, and the Enola Gay* dropped one tiny atom on Japan. The children would leave his classroom for the playground to torment the weak 15

and the smart, mussing up their hair and breaking their glasses, while he gathered up his notes and walked home past flower beds and white picket fences, wondering if they would believe that soldiers 20

in the Boer War told long, rambling stories designed to make the enemy nod off. —Billy Collins "The History Teacher" from QUESTIONS ABOUT ANGELS, by Billy Collins, © 1991. Reprinted by permission of the University of Pittsburgh Press.

*The name of the airplane from which an atomic bomb was dropped on

Hiroshima, Japan, in 1945.

Line

Line

Page 33: 2004 AP English Literature and Composition Free …bayareatutoring.com/APlitprompts.pdf · AP® English Literature and Composition 2004 Free-Response Questions Form B ... present,

2007 AP® ENGLISH LITERATURE AND COMPOSITION FREE-RESPONSE QUESTIONS

© 2007 The College Board. All rights reserved. Visit apcentral.collegeboard.com (for AP professionals) and www.collegeboard.com/apstudents (for students and parents).

GO ON TO THE NEXT PAGE. -3-

Question 2

(Suggested time—40 minutes. This question counts as one-third of the total essay section score.) Read carefully the following passage from Dalton Trumbo’s novel Johnny Got His Gun (1939). Then write a well-organized essay in which you analyze how Trumbo uses such techniques as point of view, selection of detail, and syntax to characterize the relationship between the young man and his father.

The campfire was built in front of a tent and the tent was under an enormous pine. When you slept inside the tent it seemed always that it was raining outside because the needles from the pine kept falling. Sitting across from him and staring into the fire was 5

his father. Each summer they came to this place which was nine thousand feet high and covered with pine trees and dotted with lakes. They fished in the lakes and when they slept at night the roar of water from the streams which connected the lakes sounded 10

in their ears all night long. They had been coming to this place ever since he

was seven. Now he was fifteen and Bill Harper was going to come tomorrow. He sat in front of the fire and looked across at his father and wondered just how 15

he was going to tell him. It was a very serious thing. Tomorrow for the first time in all their trips together he wanted to go fishing with someone other than his father. On previous trips the idea had never occurred to him. His father had always preferred his company 20

to that of men and he had always preferred his father’s company to that of the other guys. But now Bill Harper was coming up tomorrow and he wanted to go fishing with him. He knew it was something that had to happen sometime. Yet he also knew that it was 25

the end of something. It was an ending and a beginning and he wondered just how he should tell his father about it.

So he told him very casually. He said Bill Harper’s coming up tomorrow and I thought maybe I’d go out 30

with him. He said Bill Harper doesn’t know very much about fishing and I do so I think if you don’t

mind I’ll get up early in the morning and meet Harper and he and I will go fishing.

For a little while his father didn’t say a thing. Then 35

he said why sure go along Joe. And then a little later his father said has Bill Harper got a rod? He told his father no Bill hasn’t a rod. Well said his father why don’t you take my rod and let Bill use yours? I don’t want to go fishing tomorrow anyhow. I’m tired and I 40

think I’ll rest all day. So you use my rod and let Bill use yours.

It was as simple as that and yet he knew it was a great thing. His father’s rod was a very valuable one. It was perhaps the only extravagance his father had 45

had in his whole life. It had amber leaders and beautiful silk windings. Each spring his father sent the rod away to a man in Colorado Springs who was an expert on rods. The man in Colorado Springs carefully scraped the varnish off the rod and rewound 50

it and revarnished it and it came back glistening new each year. There was nothing his father treasured more. He felt a little lump in his throat as he thought that even as he was deserting his father for Bill Harper his father had volunteered the rod. 55

They went to sleep that night in the bed which lay against a floor of pine needles. They had scooped the needles out to make a little hollow place for their hips. He lay awake quite a while thinking about tomorrow and his father who slept beside him. Then he fell 60

asleep. At six o’clock Bill Harper whispered to him through the tent flap. He got up and gave Bill his rod and took his father’s for himself and they went off without awakening his father.

Line

Page 34: 2004 AP English Literature and Composition Free …bayareatutoring.com/APlitprompts.pdf · AP® English Literature and Composition 2004 Free-Response Questions Form B ... present,

2007 AP® ENGLISH LITERATURE AND COMPOSITION FREE-RESPONSE QUESTIONS

© 2007 The College Board. All rights reserved. Visit apcentral.collegeboard.com (for AP professionals) and www.collegeboard.com/apstudents (for students and parents).

-4-

Question 3

(Suggested time—40 minutes. This question counts as one-third of the total essay section score.) In many works of literature, past events can affect, positively or negatively, the present actions, attitudes, or values of a character. Choose a novel or play in which a character must contend with some aspect of the past, either personal or societal. Then write an essay in which you show how the character’s relationship to the past contributes to the meaning of the work as a whole. You may choose a work from the list below or another appropriate novel or play of similar literary merit. Do not merely summarize the plot. Absalom, Absalom! All the King’s Men The American Atonement The Awakening Beloved The Blind Assassin The Bonesetter’s Daughter The Cherry Orchard Cry, the Beloved Country Death of a Salesman An Enemy of the People Ethan Frome Fifth Business The Fixer Great Expectations The Great Gatsby The House of Mirth Jane Eyre The Kite Runner

Long Day’s Journey into Night Lord Jim Middlemarch Moby-Dick The Moor’s Last Sigh Mrs. Dalloway Native Speaker Obasan A Passage to India Persuasion The Piano Lesson Ragtime A Separate Peace A Streetcar Named Desire The Tempest Tess of the D’Urbervilles Their Eyes Were Watching God Who’s Afraid of Virginia Woolf? Wuthering Heights

STOP

END OF EXAM

Page 35: 2004 AP English Literature and Composition Free …bayareatutoring.com/APlitprompts.pdf · AP® English Literature and Composition 2004 Free-Response Questions Form B ... present,

AP® English Literature and Composition 2008 Free-Response Questions

Form B

The College Board: Connecting Students to College Success

The College Board is a not-for-profit membership association whose mission is to connect students to college success and

opportunity. Founded in 1900, the association is composed of more than 5,000 schools, colleges, universities, and other

educational organizations. Each year, the College Board serves seven million students and their parents, 23,000 high schools, and

3,500 colleges through major programs and services in college admissions, guidance, assessment, financial aid, enrollment, and

teaching and learning. Among its best-known programs are the SAT®

, the PSAT/NMSQT®

, and the Advanced Placement

Program®

(AP®

). The College Board is committed to the principles of excellence and equity, and that commitment is embodied

in all of its programs, services, activities, and concerns.

© 2008 The College Board. All rights reserved. College Board, Advanced Placement Program, AP, AP Central, SAT, and the

acorn logo are registered trademarks of the College Board. PSAT/NMSQT is a registered trademark of the College Board and

National Merit Scholarship Corporation.

Permission to use copyrighted College Board materials may be requested online at:

www.collegeboard.com/inquiry/cbpermit.html.

Visit the College Board on the Web: www.collegeboard.com. AP Central is the official online home for the AP Program: apcentral.collegeboard.com.

Page 36: 2004 AP English Literature and Composition Free …bayareatutoring.com/APlitprompts.pdf · AP® English Literature and Composition 2004 Free-Response Questions Form B ... present,

2008 AP® ENGLISH LITERATURE AND COMPOSITION FREE-RESPONSE QUESTIONS (Form B)

© 2008 The College Board. All rights reserved. Visit apcentral.collegeboard.com (for AP professionals) and www.collegeboard.com/apstudents (for students and parents).

GO ON TO THE NEXT PAGE. -2-

ENGLISH LITERATURE AND COMPOSITION SECTION II

Total time—2 hours

Question 1

(Suggested time—40 minutes. This question counts as one-third of the total essay section score.)

The following two poems present animal-eye views of the world. Read each poem carefully. Then write an essay in which you analyze the techniques used in the poems to characterize the speakers and convey differing views of the world.

HAWK ROOSTING I sit in the top of the wood, my eyes closed. Inaction, no falsifying dream Between my hooked head and hooked feet: Or in sleep rehearse perfect kills and eat. The convenience of the high trees! 5

The air’s buoyancy and the sun’s ray Are of advantage to me; And the earth’s face upward for my inspection. My feet are locked upon the rough bark. It took the whole of Creation 10

To produce my foot, my each feather: Now I hold Creation in my foot Or fly up, and revolve it all slowly— I kill where I please because it is all mine. There is no sophistry in my body: 15

My manners are tearing off heads— The allotment of death. For the one path of my flight is direct Through the bones of the living. No arguments assert my right: 20

The sun is behind me. Nothing has changed since I began. My eye has permitted no change. I am going to keep things like this. —Ted Hughes From Lupercal, by Ted Hughes. Faber & Faber Ltd., 1960.

GOLDEN RETRIEVALS Fetch? Balls and sticks capture my attention seconds at a time. Catch? I don’t think so. Bunny, tumbling leaf, a squirrel who’s—oh joy—actually scared. Sniff the wind, then I’m off again: muck, pond, ditch, residue 5

of any thrillingly dead thing. And you? Either you’re sunk in the past, half our walk, thinking of what you never can bring back, or else you’re off in some fog concerning —tomorrow, is that what you call it? My work: 10

to unsnare time’s warp (and woof!), retrieving, my haze-headed friend, you. This shining bark, a Zen master’s bronzy gong, calls you here, entirely, now: bow-wow, bow-wow, bow-wow. —Mark Doty Copyright © 1998 by Mark Doty. Reprinted by permission of HarperCollins Publishers, Inc.

Line Line

Page 37: 2004 AP English Literature and Composition Free …bayareatutoring.com/APlitprompts.pdf · AP® English Literature and Composition 2004 Free-Response Questions Form B ... present,

2008 AP® ENGLISH LITERATURE AND COMPOSITION FREE-RESPONSE QUESTIONS (Form B)

© 2008 The College Board. All rights reserved. Visit apcentral.collegeboard.com (for AP professionals) and www.collegeboard.com/apstudents (for students and parents).

GO ON TO THE NEXT PAGE. -3-

Question 2

(Suggested time — 40 minutes. This question counts as one-third of the total essay section score.) Jane Austen’s novel Northanger Abbey (1818) opens with the following passage. Read the passage carefully. Then, in a well-organized essay, analyze the literary techniques Austen uses to characterize Catherine Morland.

No one who had ever seen Catherine Morland in her infancy would have supposed her born to be an heroine. Her situation in life, the character of her father and mother, her own person and disposition, were all equally against her. Her father was a 5

clergyman, without being neglected or poor, and a very respectable man, though his name was Richard, and he had never been handsome. He had a considerable independence besides two good livings,1 and he was not in the least addicted to 10

locking up his daughters. Her mother was a woman of useful plain sense, with a good temper, and, what is more remarkable, with a good constitution. She had three sons before Catherine was born; and, instead of dying in bringing the latter into the world, as 15

anybody might expect, she still lived on—lived to have six children more—to see them growing up around her, and to enjoy excellent health herself. A family of ten children will be always called a fine family, where there are heads, and arms, and legs 20

enough for the number; but the Morlands had little other right to the word, for they were in general very plain, and Catherine, for many years of her life, as plain as any. She had a thin awkward figure, a sallow skin without colour, dark lank hair, and 25

strong features; so much for her person, and not less unpropitious for heroism seemed her mind. She was fond of all boys’ play and greatly preferred cricket, not merely to dolls, but to the more heroic enjoyments of infancy, nursing a dormouse, feeding a canary-bird, 30

or watering a rose-bush. Indeed she had no taste for a garden, and if she gathered flowers at all, it was chiefly for the pleasure of mischief, at least so it was conjectured from her always preferring those which she was forbidden to take. Such were her propensities; 35

her abilities were quite as extraordinary. She never could learn or understand anything before she was taught, and sometimes not even then, for she was

often inattentive, and occasionally stupid. Her mother was three months in teaching her only to repeat the 40

“Beggar’s Petition,” and, after all, her next sister Sally could say it better than she did. Not that Catherine was always stupid; by no means; she learnt the fable of “The Hare and many Friends,” as quickly as any girl in England. Her mother wished her to learn 45

music; and Catherine was sure she should like it, for she was very fond of tinkling the keys of the old forlorn spinnet,2 so at eight years old she began. She learnt a year and could not bear it; and Mrs. Morland, who did not insist on her daughters being accom-50

plished in spite of incapacity or distaste, allowed her to leave off. The day which dismissed the music-master was one of the happiest of Catherine’s life. Her taste for drawing was not superior; though whenever she could obtain the outside of a letter 55

from her mother, or seize upon any other odd piece of paper, she did what she could in that way by drawing houses and trees, hens and chickens, all very much like one another. Writing and accounts she was taught by her father; French by her mother. 60

Her proficiency in either was not remarkable, and she shirked her lessons in both whenever she could. What a strange unaccountable character! for with all these symptoms of profligacy at ten years old, she had neither a bad heart nor a bad temper, was seldom 65

stubborn, scarcely ever quarrelsome, and very kind to the little ones, with few interruptions of tyranny. She was, moreover, noisy and wild, hated confine-ment and cleanliness, and loved nothing so well in the world as rolling down the green slope at the back 70

of the house. 1 Incomes or endowments 2 Piano

Line

Page 38: 2004 AP English Literature and Composition Free …bayareatutoring.com/APlitprompts.pdf · AP® English Literature and Composition 2004 Free-Response Questions Form B ... present,

2008 AP® ENGLISH LITERATURE AND COMPOSITION FREE-RESPONSE QUESTIONS (Form B)

© 2008 The College Board. All rights reserved. Visit apcentral.collegeboard.com (for AP professionals) and www.collegeboard.com/apstudents (for students and parents).

-4-

Question 3

(Suggested time—40 minutes. This question counts as one-third of the total essay section score.) In some works of literature, childhood and adolescence are portrayed as times graced by innocence and a sense of wonder; in other works, they are depicted as times of tribulation and terror. Focusing on a single novel or play, explain how its representation of childhood or adolescence shapes the meaning of the work as a whole. You may select a work from the list below or choose another appropriate novel or play of similar literary merit. Avoid mere plot summary.

The Adventures of Huckleberry Finn Black Boy Bless Me, Ultima The Bluest Eye The Catcher in the Rye Cat’s Eye The Chosen Great Expectations A High Wind in Jamaica The House on Mango Street Jane Eyre Kafka on the Shore Little Women Lord of the Flies “Master Harold” . . . and the boys The Member of the Wedding My Ántonia

Native Speaker Old School Pocho A Portrait of the Artist As a Young Man The Prime of Miss Jean Brodie The Red Badge of Courage A River Runs Through It Romeo and Juliet Sula To Kill a Mockingbird To the Lighthouse Tom Jones The Turn of the Screw Wide Sargasso Sea Woman Warrior Wuthering Heights

STOP

END OF EXAM

Page 39: 2004 AP English Literature and Composition Free …bayareatutoring.com/APlitprompts.pdf · AP® English Literature and Composition 2004 Free-Response Questions Form B ... present,

AP® English Literature and Composition 2008 Free-Response Questions

The College Board: Connecting Students to College Success

The College Board is a not-for-profit membership association whose mission is to connect students to college success and opportunity. Founded in 1900, the association is composed of more than 5,000 schools, colleges, universities, and other educational organizations. Each year, the College Board serves seven million students and their parents, 23,000 high schools, and 3,500 colleges through major programs and services in college admissions, guidance, assessment, financial aid, enrollment, and teaching and learning. Among its best-known programs are the SAT®, the PSAT/NMSQT®, and the Advanced Placement Program® (AP®). The College Board is committed to the principles of excellence and equity, and that commitment is embodied in all of its programs, services, activities, and concerns.

© 2008 The College Board. All rights reserved. College Board, Advanced Placement Program, AP, AP Central, SAT, and the acorn logo are registered trademarks of the College Board. PSAT/NMSQT is a registered trademark of the College Board and National Merit Scholarship Corporation. Permission to use copyrighted College Board materials may be requested online at: www.collegeboard.com/inquiry/cbpermit.html. Visit the College Board on the Web: www.collegeboard.com. AP Central is the official online home for the AP Program: apcentral.collegeboard.com.

Page 40: 2004 AP English Literature and Composition Free …bayareatutoring.com/APlitprompts.pdf · AP® English Literature and Composition 2004 Free-Response Questions Form B ... present,

2008 AP® ENGLISH LITERATURE AND COMPOSITION FREE-RESPONSE QUESTIONS

© 2008 The College Board. All rights reserved. Visit apcentral.collegeboard.com (for AP professionals) and www.collegeboard.com/apstudents (for students and parents).

GO ON TO THE NEXT PAGE. -2-

ENGLISH LITERATURE AND COMPOSITION SECTION II

Total time—2 hours

Question 1

(Suggested time—40 minutes. This question counts as one-third of the total essay section score.)

In the two poems below, Keats and Longfellow reflect on similar concerns. Read the poems carefully. Then write an essay in which you compare and contrast the two poems, analyzing the poetic techniques each writer uses to explore his particular situation.

When I Have Fears

When I have fears that I may cease to be Before my pen has glean’d my teeming brain, Before high-piled books, in charactery, Hold like rich garners the full ripen’d grain; When I behold, upon the night’s starr’d face, 5

Huge cloudy symbols of a high romance, And think that I may never live to trace Their shadows, with the magic hand of chance; And when I feel, fair creature of an hour, That I shall never look upon thee more, 10

Never have relish in the faery power Of unreflecting love;—then on the shore Of the wide world I stand alone, and think Till love and fame to nothingness do sink.

1818 —John Keats (1795-1821)

Mezzo Cammin1

Written at Boppard on the Rhine August 25, 1842,

Just Before Leaving for Home Half of my life is gone, and I have let The years slip from me and have not fulfilled The aspiration of my youth, to build Some tower of song with lofty parapet. Not indolence, nor pleasure, nor the fret 5

Of restless passions that would not be stilled, But sorrow, and a care that almost killed, Kept me from what I may accomplish yet; Though, half-way up the hill, I see the Past Lying beneath me with its sounds and sights,— 10

A city in the twilight dim and vast, With smoking roofs, soft bells, and gleaming lights,— And hear above me on the autumnal blast The cataract2 of Death far thundering from the heights.

1842 —Henry Wadsworth Longfellow (1807-1882)

1 The title is from the first line of Dante’s Divine Comedy: “Nel mezzo del cammin di nostra vita” (“Midway upon the journey of our life”).

2 A large waterfall

Line Line

Page 41: 2004 AP English Literature and Composition Free …bayareatutoring.com/APlitprompts.pdf · AP® English Literature and Composition 2004 Free-Response Questions Form B ... present,

2008 AP® ENGLISH LITERATURE AND COMPOSITION FREE-RESPONSE QUESTIONS

© 2008 The College Board. All rights reserved. Visit apcentral.collegeboard.com (for AP professionals) and www.collegeboard.com/apstudents (for students and parents).

GO ON TO THE NEXT PAGE. -3-

Question 2

(Suggested time—40 minutes. This question counts as one-third of the total essay section score.) The following passage is taken from Fasting, Feasting, a novel published in 1999 by Indian novelist Anita Desai. In the excerpt, Arun, an exchange student from India, joins members of his American host family for an afternoon at the beach. Read the passage carefully. Then write an essay in which you analyze how the author uses such literary devices as speech and point of view to characterize Arun’s experience.

It is Saturday. Arun cannot plead work. He stands despondent, and when Melanie comes to the door, dressed in her bathing suit with a big shirt drawn over her shoulders, and stares at him challengingly, he starts wildly to find excuses. 5

Mrs. Patton will not hear them. No, she will not. Absolutely not. So she says, with her hands spread out and pressing against the air. ‘No, no, no. We’re all three of us going. Rod and Daddy have gone sailing on Lake Wyola and we’re not going to sit here 10

waiting for them to come home—oh no.’ Arun must go back upstairs and collect his towel

and swimming trunks. Then he follows Melanie to the driveway where Mrs. Patton is waiting with baskets of equipment—oils and lotions, paperbacks and dark 15

glasses, sandwiches and lemonade. With that new and animated prance galvanising her dwindled shanks, she leads the way through a gap in the bushes to one of the woodland paths. Melanie and Arun follow silently. They try to find a way to walk that will not 20

compel them to be side by side or in any way close together. But who is to follow whom? It is an awkward problem. Arun finally stops trying to lag behind her—she can lag even better—and goes ahead to catch up with Mrs. Patton. He ought to help 25

carry those baskets anyway. He takes one from her hands and she throws him a radiant, lipsticked smile. Then she swings away and goes confidently forwards.

‘Summertime,’ he hears her singing, ‘when the living is eeh-zee—’ 30

They make their way along scuffed paths through layers of old soft pine needles. The woods are thrumming with cicadas: they shrill and shrill as if the sun is playing on their sinews, as if they were small harps suspended in the trees. A bird shrieks 35

hoarsely, flies on, shrieks elsewhere, further off—that ugly, jarring note that does not vary. But there are no birds to be seen, nor animals. It is as if they are in hiding, or have fled. Perhaps they have because the houses of Edge Hill do intrude and one can glimpse 40

a bit of wall here or roof there, a washing line hung with sheets or a plastic gnome, finger to nose, enigmatically winking. Arun finds the hair on the back of his neck begin to prickle, as if in warning. He is sweating, and the palms of his hands are becoming 45

puffy and damp. Why must people live in the vicinity of such benighted wilderness and become a part of it? The town may be small and have little to offer, but how passionately he prefers its post office, its shops, its dry-cleaning stores and picture framers to this 50

creeping curtain of insidious green, these grasses stirring with insidious life, and bushes with poisonous berries—so bright or else so pale. Nearly tripping upon a root, he stumbles and has to steady himself so as not to spill the contents of the basket. 55

Line

Page 42: 2004 AP English Literature and Composition Free …bayareatutoring.com/APlitprompts.pdf · AP® English Literature and Composition 2004 Free-Response Questions Form B ... present,

2008 AP® ENGLISH LITERATURE AND COMPOSITION FREE-RESPONSE QUESTIONS

© 2008 The College Board. All rights reserved. Visit apcentral.collegeboard.com (for AP professionals) and www.collegeboard.com/apstudents (for students and parents).

-4-

Question 3

(Suggested time—40 minutes. This question counts as one-third of the total essay section score.) In a literary work, a minor character, often known as a foil, possesses traits that emphasize, by contrast or comparison, the distinctive characteristics and qualities of the main character. For example, the ideas or behavior of the minor character might be used to highlight the weaknesses or strengths of the main character. Choose a novel or play in which a minor character serves as a foil to a main character. Then write an essay in which you analyze how the relation between the minor character and the major character illuminates the meaning of the work. You may choose a work from the list below or another appropriate novel or play of similar literary quality. Do not merely summarize the plot. The Age of Innocence Alias Grace All the King’s Men All the Pretty Horses Anna Karenina Billy Budd The Brothers Karamazov Catch-22 Cold Mountain The Color Purple Don Quixote Emma Equus Frankenstein Glass Menagerie Henry IV, Part I

Huckleberry Finn Invisible Man King Lear The Kite Runner The Misanthrope The Piano Lesson Pride and Prejudice Pygmalion Reservation Blues The Sound and the Fury A Streetcar Named Desire Sula A Tale of Two Cities Their Eyes Were Watching God Tom Jones Wuthering Heights

STOP

END OF EXAM

Page 43: 2004 AP English Literature and Composition Free …bayareatutoring.com/APlitprompts.pdf · AP® English Literature and Composition 2004 Free-Response Questions Form B ... present,

AP® English Literature and Composition 2009 Free-Response Questions

Form B

The College Board

The College Board is a not-for-profit membership association whose mission is to connect students to college success and opportunity. Founded in 1900, the association is composed of more than 5,600 schools, colleges, universities and other educational organizations. Each year, the College Board serves seven million students and their parents, 23,000 high schools and 3,800 colleges through major programs and services in college readiness, college admissions, guidance, assessment, financial aid, enrollment, and teaching and learning. Among its best-known programs are the SAT®, the PSAT/NMSQT® and the Advanced Placement Program® (AP®). The College Board is committed to the principles of excellence and equity, and that commitment is embodied in all of its programs, services, activities and concerns.

© 2009 The College Board. All rights reserved. College Board, Advanced Placement Program, AP, AP Central, SAT, and the acorn logo are registered trademarks of the College Board. PSAT/NMSQT is a registered trademark of the College Board and National Merit Scholarship Corporation. Permission to use copyrighted College Board materials may be requested online at: www.collegeboard.com/inquiry/cbpermit.html. Visit the College Board on the Web: www.collegeboard.com. AP Central is the official online home for the AP Program: apcentral.collegeboard.com.

Page 44: 2004 AP English Literature and Composition Free …bayareatutoring.com/APlitprompts.pdf · AP® English Literature and Composition 2004 Free-Response Questions Form B ... present,

2009 AP® ENGLISH LITERATURE AND COMPOSITION FREE-RESPONSE QUESTIONS (Form B)

© 2009 The College Board. All rights reserved. Visit the College Board on the Web: www.collegeboard.com.

GO ON TO THE NEXT PAGE. -2-

ENGLISH LITERATURE AND COMPOSITION SECTION II

Total time—2 hours Question 1

(Suggested time—40 minutes. This question counts as one-third of the total essay section score.) The following poem, written by Edward Field, makes use of the Greek myth of Daedalus and Icarus.* Read the poem carefully. Then write an essay in which you analyze how Field employs literary devices in adapting the Icarus myth to a contemporary setting.

Icarus Only the feathers floating around the hat Showed that anything more spectacular had occurred Than the usual drowning. The police preferred to ignore The confusing aspects of the case, And the witnesses ran off to a gang war. 5

So the report filed and forgotten in the archives read simply “Drowned,” but it was wrong: Icarus Had swum away, coming at last to the city Where he rented a house and tended the garden. “That nice Mr. Hicks” the neighbors called him, 10

Never dreaming that the gray, respectable suit Concealed arms that had controlled huge wings Nor that those sad, defeated eyes had once Compelled the sun. And had he told them They would have answered with a shocked, uncomprehending stare. 15

No, he could not disturb their neat front yards; Yet all his books insisted that this was a horrible mistake: What was he doing aging in a suburb? Can the genius of the hero fall To the middling stature of the merely talented? 20

And nightly Icarus probes his wound And daily in his workshop, curtains carefully drawn, Constructs small wings and tries to fly To the lighting fixture on the ceiling: Fails every time and hates himself for trying. 25

He had thought himself a hero, had acted heroically, And dreamt of his fall, the tragic fall of the hero; But now rides commuter trains, Serves on various committees, And wishes he had drowned. 30 Reprinted by permission of the author.

* Daedalus and his son, Icarus, fashioned wings of feathers and wax in an attempt to escape from prison by flying across the sea. Before their flight, Daedalus

warned his son not to fly too close to the sun. But, caught up in the experience of flying, Icarus ignored the warning and soared upward. The heat of the sun melted the wax, the wings fell off, and he plunged to his death in the sea.

Line

Page 45: 2004 AP English Literature and Composition Free …bayareatutoring.com/APlitprompts.pdf · AP® English Literature and Composition 2004 Free-Response Questions Form B ... present,

2009 AP® ENGLISH LITERATURE AND COMPOSITION FREE-RESPONSE QUESTIONS (Form B)

© 2009 The College Board. All rights reserved. Visit the College Board on the Web: www.collegeboard.com.

GO ON TO THE NEXT PAGE. -3-

Question 2

(Suggested time—40 minutes. This question counts as one-third of the total essay section score.) The passage below is the opening of Seraph on the Suwanee (1948), a novel written by Zora Neale Hurston. Read the passage carefully. Then write a well-organized essay in which you analyze the literary techniques Hurston uses to describe Sawley and to characterize the people who live there.

Sawley, the town, is in west Florida, on the famous

Suwanee River. It is flanked on the south by the curving course of the river which Stephen Foster* made famous without ever having looked upon its waters, running swift and deep through the primitive 5

forests, and reddened by the chemicals leeched out of drinking roots. On the north, the town is flanked by cultivated fields planted to corn, cane, potatoes, tobacco and small patches of cotton.

However, few of these fields were intensively 10

cultivated. For the most part they were scratchy plantings, the people being mostly occupied in the production of turpentine and lumber. The life of Sawley streamed out from the sawmill and the “teppentime ‘still.” Then too, there was ignorance 15

and poverty, and the ever-present hookworm. The farms and the scanty flowers in front yards and in tin cans and buckets looked like the people. Trees and plants always look like the people they live with, somehow. 20

This was in the first decade of the new century, when the automobile was known as the horseless carriage, and had not exerted its tremendous influence on the roads of the nation. There was then no U.S. 90, the legendary Old Spanish Trail, stretching straight 25

broad concrete from Jacksonville on the Atlantic to San Diego on the Pacific. There was the sandy pike, deeply rutted by wagon wheels over which the folks of Sawley hauled their tobacco to market at Live Oak, or fresh-killed hogmeat, corn and peanuts to Madison 30

or Monticello on the west. Few ever dreamed of venturing any farther east nor west.

Few were concerned with the past. They had heard that the stubbornly resisting Indians had been there where they now lived, but they were dead and gone. 35

Osceola, Miccanope, Billy Bow-Legs were nothing more than names that had even lost their bitter flavor. The conquering Spaniards had done their murdering, robbing, and raping and had long ago withdrawn from the Floridas. Few knew and nobody cared that the 40

Hidalgos under De Sota had moved westward along this very route. The people thought no more of them than they did the magnolias and bay and other ornamental trees which grew so plentifully in the swamps along the river, nor the fame of the stream. 45

They knew that there were plenty of black bass, locally known as trout, in the Suwanee, and bream and perch and cat-fish. There were soft-shell turtles that made a mighty nice dish when stewed down to a low gravy, or the “chicken meat” of those same 50

turtles fried crisp and brown. Fresh water turtles were a mighty fine article of food anyway you looked at it. It was commonly said that a turtle had every kind of meat on him. The white “chicken meat,” the dark “beef” and the in-between “pork.” You could stew, 55

boil and fry, and none of it cost you a cent. All you needed was a strip of white side-meat on the hook, and you had you some turtle meat. *American songwriter (1826-1864) whose song “Old Folks at Home”

begins “Way down upon the Swanee River”

Line

Page 46: 2004 AP English Literature and Composition Free …bayareatutoring.com/APlitprompts.pdf · AP® English Literature and Composition 2004 Free-Response Questions Form B ... present,

2009 AP® ENGLISH LITERATURE AND COMPOSITION FREE-RESPONSE QUESTIONS (Form B)

© 2009 The College Board. All rights reserved. Visit the College Board on the Web: www.collegeboard.com.

-4-

Question 3

(Suggested time—40 minutes. This question counts as one-third of the total essay section score.) Many works of literature deal with political or social issues. Choose a novel or play that focuses on a political or social issue. Then write an essay in which you analyze how the author uses literary elements to explore this issue and explain how the issue contributes to the meaning of the work as a whole. Do not merely summarize the plot. You may choose a work from the list below or another novel or play of similar literary merit.

All the King’s Men American Pastoral Angels in America Anna Karenina Antigone The Awakening The Bluest Eye Brave New World Broken for You Ceremony Copenhagen The Crucible Cry, the Beloved Country A Doll House Equus Germinal Go Tell It on the Mountain The Grapes of Wrath Gulliver’s Travels The Handmaid’s Tale Hard Times House Made of Dawn Invisible Man Jude the Obscure

Julius Caesar The Jungle Main Street Major Barbara “Master Harold” . . . and the Boys Moll Flanders Mrs. Warren’s Profession Native Son Never Let Me Go 1984 The Octopus Oliver Twist A Passage to India A Raisin in the Sun Reservation Blues Rhinoceros Sister Carrie Sophie’s Choice The Things They Carried To Kill a Mockingbird Uncle Tom’s Cabin USA The Women of Brewster Place

STOP

END OF EXAM

Page 47: 2004 AP English Literature and Composition Free …bayareatutoring.com/APlitprompts.pdf · AP® English Literature and Composition 2004 Free-Response Questions Form B ... present,

AP® English Literature and Composition 2009 Free-Response Questions

The College Board

The College Board is a not-for-profit membership association whose mission is to connect students to college success and opportunity. Founded in 1900, the association is composed of more than 5,600 schools, colleges, universities and other educational organizations. Each year, the College Board serves seven million students and their parents, 23,000 high schools and 3,800 colleges through major programs and services in college readiness, college admissions, guidance, assessment, financial aid, enrollment, and teaching and learning. Among its best-known programs are the SAT®, the PSAT/NMSQT® and the Advanced Placement Program® (AP®). The College Board is committed to the principles of excellence and equity, and that commitment is embodied in all of its programs, services, activities and concerns.

© 2009 The College Board. All rights reserved. College Board, Advanced Placement Program, AP, AP Central, SAT, and the acorn logo are registered trademarks of the College Board. PSAT/NMSQT is a registered trademark of the College Board and National Merit Scholarship Corporation. Permission to use copyrighted College Board materials may be requested online at: www.collegeboard.com/inquiry/cbpermit.html. Visit the College Board on the Web: www.collegeboard.com. AP Central is the official online home for the AP Program: apcentral.collegeboard.com.

Page 48: 2004 AP English Literature and Composition Free …bayareatutoring.com/APlitprompts.pdf · AP® English Literature and Composition 2004 Free-Response Questions Form B ... present,

2009 AP® ENGLISH LITERATURE AND COMPOSITION FREE-RESPONSE QUESTIONS

© 2009 The College Board. All rights reserved. Visit the College Board on the Web: www.collegeboard.com.

GO ON TO THE NEXT PAGE. -2-

ENGLISH LITERATURE AND COMPOSITION SECTION II

Total time—2 hours

Question 1

(Suggested time—40 minutes. This question counts as one-third of the total essay section score.) In the following speech from Shakespeare’s play Henry VIII, Cardinal Wolsey considers his sudden downfall from his position as advisor to the king. Spokesmen for the king have just left Wolsey alone on stage. Read the speech carefully. Then write a well-organized essay in which you analyze how Shakespeare uses elements such as allusion, figurative language, and tone to convey Wolsey’s complex response to his dismissal from court.

So farewell—to the little good you bear me. Farewell? a long farewell to all my greatness! This is the state of man: to-day he puts forth The tender leaves of hopes, to-morrow blossoms, And bears his blushing honors thick upon him; 5

The third day comes a frost, a killing frost, And when he thinks, good easy man, full surely His greatness is a-ripening, nips his root, And then he falls as I do. I have ventur’d, Like little wanton boys that swim on bladders,1 10

This many summers in a sea of glory, But far beyond my depth. My high-blown pride At length broke under me, and now has left me, Weary and old with service, to the mercy Of a rude stream that must for ever hide me. 15

Vain pomp and glory of this world, I hate ye! I feel my heart new open’d. O how wretched Is that poor man that hangs on princes’ favors! There is, betwixt that smile we would aspire to, That sweet aspect of princes, and their ruin, 20

More pangs and fears than wars or women have; And when he falls, he falls like Lucifer,2 Never to hope again. 1 air-filled sacs 2 Satan, the fallen angel

Line

Page 49: 2004 AP English Literature and Composition Free …bayareatutoring.com/APlitprompts.pdf · AP® English Literature and Composition 2004 Free-Response Questions Form B ... present,

2009 AP® ENGLISH LITERATURE AND COMPOSITION FREE-RESPONSE QUESTIONS

© 2009 The College Board. All rights reserved. Visit the College Board on the Web: www.collegeboard.com.

GO ON TO THE NEXT PAGE. -3-

Question 2

(Suggested time—40 minutes. This question counts as one-third of the total essay section score.) The following selection is the opening of Ann Petry’s 1946 novel, The Street. Read the selection carefully and then write an essay analyzing how Petry establishes Lutie Johnson’s relationship to the urban setting through the use of such literary devices as imagery, personification, selection of detail, and figurative language.

There was a cold November wind blowing through

116th Street. It rattled the tops of garbage cans, sucked window shades out through the top of opened windows and set them flapping back against the windows; and it drove most of the people off the 5

street in the block between Seventh and Eighth Avenues except for a few hurried pedestrians who bent double in an effort to offer the least possible exposed surface to its violent assault.

It found every scrap of paper along the street—10

theater throwaways, announcements of dances and lodge meetings, the heavy waxed paper that loaves of bread had been wrapped in, the thinner waxed paper that had enclosed sandwiches, old envelopes, newspapers. Fingering its way along the curb, the 15

wind set the bits of paper to dancing high in the air, so that a barrage of paper swirled into the faces of the people on the street. It even took time to rush into doorways and areaways and find chicken bones and pork-chop bones and pushed them along the curb. 20

It did everything it could to discourage the people walking along the street. It found all the dirt and dust and grime on the sidewalk and lifted it up so that the dirt got into their noses, making it difficult to breathe; the dust got into their eyes and blinded them; and the 25

grit stung their skins. It wrapped newspaper around their feet entangling them until the people cursed deep in their throats, stamped their feet, kicked at the paper. The wind blew it back again and again until they were forced to stoop and dislodge the paper with 30

their hands. And then the wind grabbed their hats,

pried their scarves from around their necks, stuck its fingers inside their coat collars, blew their coats away from their bodies.

The wind lifted Lutie Johnson’s hair away from the 35

back of her neck so that she felt suddenly naked and bald, for her hair had been resting softly and warmly against her skin. She shivered as the cold fingers of the wind touched the back of her neck, explored the sides of her head. It even blew her eyelashes away 40

from her eyes so that her eyeballs were bathed in a rush of coldness and she had to blink in order to read the words on the sign swaying back and forth over her head.

Each time she thought she had the sign in focus, 45

the wind pushed it away from her so that she wasn’t certain whether it said three rooms or two rooms. If it was three, why, she would go in and ask to see it, but if it said two—why, there wasn’t any point. Even with the wind twisting the sign away from her, she 50

could see that it had been there for a long time because its original coat of white paint was streaked with rust where years of rain and snow had finally eaten the paint off down to the metal and the metal had slowly rusted, making a dark red stain like blood. 55

It was three rooms. The wind held it still for an instant in front of her and then swooped it away until it was standing at an impossible angle on the rod that suspended it from the building. She read it rapidly. Three rooms, steam heat, parquet floors, respectable 60

tenants. Reasonable.

Line

Page 50: 2004 AP English Literature and Composition Free …bayareatutoring.com/APlitprompts.pdf · AP® English Literature and Composition 2004 Free-Response Questions Form B ... present,

2009 AP® ENGLISH LITERATURE AND COMPOSITION FREE-RESPONSE QUESTIONS

© 2009 The College Board. All rights reserved. Visit the College Board on the Web: www.collegeboard.com.

-4-

Question 3

(Suggested time—40 minutes. This question counts as one-third of the total essay section score.) A symbol is an object, action, or event that represents something or that creates a range of associations beyond itself. In literary works a symbol can express an idea, clarify meaning, or enlarge literal meaning. Select a novel or play and, focusing on one symbol, write an essay analyzing how that symbol functions in the work and what it reveals about the characters or themes of the work as a whole. Do not merely summarize the plot. You may choose a work from the list below or another novel or play of comparable literary merit. As I Lay Dying The Awakening Beloved Bleak House Cat’s Eye The Cherry Orchard The Color Purple Crime and Punishment The Crossing The Crucible A Doll House Equus A Farewell to Arms Fences The Glass Menagerie The Golden Bowl The Grapes of Wrath The Hairy Ape Heart of Darkness Invisible Man Jude the Obscure

The Kite Runner Lady Windermere’s Fan Macbeth Madame Bovary The Memory Keeper’s Daughter Moby-Dick The Namesake Nineteen Eighty-four Our Town The Plague A Portrait of the Artist as a Young Man A Prayer for Owen Meany A Raisin in the Sun Reservation Blues Snow A Streetcar Named Desire Things Fall Apart Waiting for Godot Wise Blood The Women of Brewster Place

STOP

END OF EXAM

Page 51: 2004 AP English Literature and Composition Free …bayareatutoring.com/APlitprompts.pdf · AP® English Literature and Composition 2004 Free-Response Questions Form B ... present,

AP® English Literature and Composition 2010 Free-Response Questions

Form B

The College Board

The College Board is a not-for-profit membership association whose mission is to connect students to college success and opportunity. Founded in 1900, the College Board is composed of more than 5,700 schools, colleges, universities and other educational organizations. Each year, the College Board serves seven million students and their parents, 23,000 high schools, and 3,800 colleges through major programs and services in college readiness, college admission, guidance, assessment, financial aid and enrollment. Among its widely recognized programs are the SAT®, the PSAT/NMSQT®, the Advanced Placement Program®

(AP®), SpringBoard® and ACCUPLACER®. The College Board is committed to the principles of excellence and equity, and that commitment is embodied in all of its programs, services, activities and concerns. © 2010 The College Board. College Board, ACCUPLACER, Advanced Placement Program, AP, AP Central, SAT, SpringBoard and the acorn logo are registered trademarks of the College Board. Admitted Class Evaluation Service is a trademark owned by the College Board. PSAT/NMSQT is a registered trademark of the College Board and National Merit Scholarship Corporation. All other products and services may be trademarks of their respective owners. Permission to use copyrighted College Board materials may be requested online at: www.collegeboard.com/inquiry/cbpermit.html. Visit the College Board on the Web: www.collegeboard.com. AP Central is the official online home for the AP Program: apcentral.collegeboard.com.

Page 52: 2004 AP English Literature and Composition Free …bayareatutoring.com/APlitprompts.pdf · AP® English Literature and Composition 2004 Free-Response Questions Form B ... present,

2010 AP® ENGLISH LITERATURE AND COMPOSITION FREE-RESPONSE QUESTIONS (Form B)

© 2010 The College Board. Visit the College Board on the Web: www.collegeboard.com.

GO ON TO THE NEXT PAGE. -2-

ENGLISH LITERATURE AND COMPOSITION SECTION II

Total time—2 hours

Question 1

(Suggested time—40 minutes. This question counts as one-third of the total essay section score.)

Each of the two poems below is concerned with a young man at the age of twenty-one, traditionally the age of adulthood. Read the two poems carefully. Then write a well-organized essay in which you compare and contrast the poems, analyzing the poetic techniques, such as point of view and tone, that each writer uses to make his point about coming of age. To Sir John Lade, on His Coming of Age (‘A Short Song of Congratulation’)

Long-expected one and twenty Lingering year at last is flown, Pomp and pleasure, pride and plenty, Great Sir John, are all your own.

Loosened from the minor’s tether, 5

Free to mortgage or to sell, Wild as wind, and light as feather, Bid the slaves of thrift farewell.

Call the Bettys, Kates, and Jennys, Every name that laughs at care, 10

Lavish of your grandsire’s guineas, Show the spirit of an heir.

All that prey on vice and folly Joy to see their quarry fly, Here the gamester light and jolly, 15

There the lender grave and sly.

Wealth, Sir John, was made to wander, Let it wander as it will; See the jockey, see the pander, Bid them come, and take their fill. 20

When the bonny blade carouses, Pockets full, and spirits high, What are acres? What are houses? Only dirt, or wet or dry.

If the guardian or the mother 25

Tell the woes of wilful waste, Scorn their counsel and their pother,* You can hang or drown at last.

1780 —Samuel Johnson (1709–1784)

* fuss

When I Was One-and-Twenty

When I was one-and-twenty I heard a wise man say, ‘Give crowns and pounds and guineas But not your heart away; Give pearls away and rubies 5

But keep your fancy free.’ But I was one-and-twenty, No use to talk to me.

When I was one-and-twenty I heard him say again, 10

‘The heart out of the bosom Was never given in vain; ’Tis paid with sighs a plenty And sold for endless rue.’ And I am two-and-twenty, 15

And oh, ’tis true, ’tis true. 1896 —A. E. Housman (1859–1936)

Line

Line

Page 53: 2004 AP English Literature and Composition Free …bayareatutoring.com/APlitprompts.pdf · AP® English Literature and Composition 2004 Free-Response Questions Form B ... present,

2010 AP® ENGLISH LITERATURE AND COMPOSITION FREE-RESPONSE QUESTIONS (Form B)

© 2010 The College Board. Visit the College Board on the Web: www.collegeboard.com.

GO ON TO THE NEXT PAGE. -3-

Question 2

(Suggested time—40 minutes. This question counts as one-third of the total essay section score.)

The following passage is taken from the story “Cherry Bomb” by Maxine Clair. Read the passage carefully. Then write an essay in which you analyze how Clair uses literary techniques to characterize the adult narrator’s memories of her fifth-grade summer world.

It was two summers before I would put my thin-

penny bus token in the slot and ride the Fifth Street trolley all the way to the end of the line to junior high. Life was measured in summers then, and the expression “I am in this world, but not of it” appealed 5

to me. I wasn’t sure what it meant, but it had just the right ring for a lofty statement I should adopt. That Midwest summer broke records for straight over-one-hundred-degree days in July, and Mr. Calhoun still came around with that-old-thing of an ice truck. Our 10

mother still bought a help-him-out block of ice to leave in the backyard for us to lick or sit on. It was the summer that the Bible’s plague of locusts came. Evening sighed its own relief in a locust hum that swelled from the cattails next to the cemetery, from 15

the bridal wreath shrubs and the pickle grass that my younger cousin, Bea, combed and braided on our side of the alley.

I kept a cherry bomb and a locked diary in the closet under the back steps where Bea, restrained by 20

my suggestion that the Hairy Man hid there, wouldn’t try to find them. It was an established, Daddy-said-so fact that at night the Hairy Man went anywhere he wanted to go but in the daytime he stayed inside the yellow house on Sherman Avenue near our school. 25

During the school year if we were so late that the patrol boys had gone inside, we would see him in his fenced-in yard, wooly-headed and bearded, hollering things we dared not repeat until a nurse kind of woman in a bandanna came out and took him back 30

inside the house with the windows painted light blue, which my mother said was a peaceful color for somebody shell-shocked.

If you parted the heavy coats between the raggedy mouton that once belonged to my father’s mother, 35

who, my father said, was his Heart when she died, and the putrid-colored jacket my father wore when he got shipped out to the dot in the Pacific Ocean where, he said, the women wore one piece of cloth and looked as fine as wine in the summertime, you would find 40

yourself right in the middle of our cave-dark closet. Then, if you closed your eyes, held your hands up over your head, placed one foot in front of the other, walked until the tips of your fingers touched the smooth cool of slanted plaster all the way down to 45

where you had to slue your feet and walk squat-legged, fell to your knees and felt around on the floor—then you would hit the strong-smelling cigar box. My box of private things.

From time to time my cousins Bea and Eddy stayed 50

with us, and on the Fourth of July the year before, Eddy had lit a cherry bomb in a Libby’s corn can and tried to lob it over the house into the alley. Before it reached the top of the porch it went off, and a piece of tin shot God-is-whipping-you straight for Eddy’s 55

eye. By the time school started that year, Eddy had a keloid* like a piece of twine down the side of his face and a black patch he had to wear until he got his glass eye that stared in a fixed angle at the sky. Nick, Eddy’s friend, began calling Eddy “Black-Eyed Pea.” 60

After Eddy’s accident, he gave me a cherry bomb. His last. I kept it in my cigar box as a sort of memento of good times. Even if I had wanted to explode it, my mother had threatened to do worse to us if we so much as looked at fireworks again. Except for 65

Christmas presents, it was the first thing anybody ever gave me. * a thick scar

Line

Page 54: 2004 AP English Literature and Composition Free …bayareatutoring.com/APlitprompts.pdf · AP® English Literature and Composition 2004 Free-Response Questions Form B ... present,

2010 AP® ENGLISH LITERATURE AND COMPOSITION FREE-RESPONSE QUESTIONS (Form B)

© 2010 The College Board. Visit the College Board on the Web: www.collegeboard.com.

-4-

Question 3

(Suggested time—40 minutes. This question counts as one-third of the total essay section score.)

“You can leave home all you want, but home will never leave you.” —Sonsyrea Tate

Sonsyrea Tate’s statement suggests that “home” may be conceived of as a dwelling, a place, or a state of mind. It may have positive or negative associations, but in either case, it may have a considerable influence on an individual. Choose a novel or play in which a central character leaves home yet finds that home remains significant. Write a well-developed essay in which you analyze the importance of “home” to this character and the reasons for its continuing influence. Explain how the character’s idea of home illuminates the larger meaning of the work. Choose a work from the list below or another appropriate novel or play of comparable literary merit. Do not merely summarize the plot. Absalom, Absalom! All the Pretty Horses Beloved Bleak House Candide The Cherry Orchard The Country of the Pointed Firs Fences A Free Life: A Novel The Glass Menagerie The God of Small Things Going After Cacciato The Grapes of Wrath Great Expectations The Great Gatsby Home to Harlem A House for Mr. Biswas The House of Mirth The House on Mango Street

The Inheritance of Loss Invisible Man Jane Eyre The Little Foxes Look Homeward, Angel The Namesake Never Let Me Go The Piano Lesson The Poisonwood Bible A Portrait of the Artist as a Young Man The Road Song of Solomon A Streetcar Named Desire Sula Their Eyes Were Watching God Things Fall Apart Wise Blood The Women of Brewster Place Wuthering Heights

STOP

END OF EXAM

Page 55: 2004 AP English Literature and Composition Free …bayareatutoring.com/APlitprompts.pdf · AP® English Literature and Composition 2004 Free-Response Questions Form B ... present,

AP® English Literature and Composition 2010 Free-Response Questions

The College Board

The College Board is a not-for-profit membership association whose mission is to connect students to college success and opportunity. Founded in 1900, the College Board is composed of more than 5,700 schools, colleges, universities and other educational organizations. Each year, the College Board serves seven million students and their parents, 23,000 high schools, and 3,800 colleges through major programs and services in college readiness, college admission, guidance, assessment, financial aid and enrollment. Among its widely recognized programs are the SAT®, the PSAT/NMSQT®, the Advanced Placement Program®

(AP®), SpringBoard® and ACCUPLACER®. The College Board is committed to the principles of excellence and equity, and that commitment is embodied in all of its programs, services, activities and concerns. © 2010 The College Board. College Board, ACCUPLACER, Advanced Placement Program, AP, AP Central, SAT, SpringBoard and the acorn logo are registered trademarks of the College Board. Admitted Class Evaluation Service is a trademark owned by the College Board. PSAT/NMSQT is a registered trademark of the College Board and National Merit Scholarship Corporation. All other products and services may be trademarks of their respective owners. Permission to use copyrighted College Board materials may be requested online at: www.collegeboard.com/inquiry/cbpermit.html. Visit the College Board on the Web: www.collegeboard.com. AP Central is the official online home for the AP Program: apcentral.collegeboard.com.

Page 56: 2004 AP English Literature and Composition Free …bayareatutoring.com/APlitprompts.pdf · AP® English Literature and Composition 2004 Free-Response Questions Form B ... present,

2010 AP® ENGLISH LITERATURE AND COMPOSITION FREE-RESPONSE QUESTIONS

© 2010 The College Board. Visit the College Board on the Web: www.collegeboard.com.

GO ON TO THE NEXT PAGE. -2-

ENGLISH LITERATURE AND COMPOSITION SECTION II

Total time—2 hours

Question 1

(Suggested time—40 minutes. This question counts as one-third of the total essay section score.) Read carefully the following poem by Marilyn Nelson Waniek. Then write an essay analyzing how Waniek employs literary techniques to develop the complex meanings that the speaker attributes to The Century Quilt. You may wish to consider such elements as structure, imagery, and tone.

The Century Quilt

for Sarah Mary Taylor, Quilter

My sister and I were in love with Meema’s Indian blanket. We fell asleep under army green issued to Daddy by Supply. When Meema came to live with us 5

she brought her medicines, her cane, and the blanket I found on my sister’s bed the last time I visited her. I remembered how I’d planned to inherit that blanket, how we used to wrap ourselves 10

at play in its folds and be chieftains and princesses.

Now I’ve found a quilt1 I’d like to die under; Six Van Dyke brown squares, 15

two white ones, and one square the yellowbrown of Mama’s cheeks. Each square holds a sweet gum leaf whose fingers I imagine would caress me into the silence. 20

I think I’d have good dreams for a hundred years under this quilt, as Meema must have, under her blanket, dreamed she was a girl again in Kentucky

among her yellow sisters, 25

their grandfather’s white family nodding at them when they met. When their father came home from his store they cranked up the pianola and all of the beautiful sisters 30

giggled and danced. She must have dreamed about Mama when the dancing was over: a lanky girl trailing after her father through his Oklahoma field. 35

Perhaps under this quilt I’d dream of myself, of my childhood of miracles, of my father’s burnt umber2 pride, my mother’s ochre3 gentleness. 40

Within the dream of myself perhaps I’d meet my son or my other child, as yet unconceived. I’d call it The Century Quilt, after its pattern of leaves. 45

Reprinted by permission of Louisiana State University Press from Mama’s Promises by Marilyn Nelson Waniek. Copyright © 1985 by Marilyn Nelson Waniek.

1 A quilt is a type of bedcovering often made by stitching together varied pieces of fabric. 2 Burnt umber is a shade of brown. 3 Ochre refers to a shade of yellow.

Line

Page 57: 2004 AP English Literature and Composition Free …bayareatutoring.com/APlitprompts.pdf · AP® English Literature and Composition 2004 Free-Response Questions Form B ... present,

2010 AP® ENGLISH LITERATURE AND COMPOSITION FREE-RESPONSE QUESTIONS

© 2010 The College Board. Visit the College Board on the Web: www.collegeboard.com.

GO ON TO THE NEXT PAGE. -3-

Question 2

(Suggested time—40 minutes. This question counts as one-third of the total essay section score.) In the following passage from Maria Edgeworth’s 1801 novel, Belinda, the narrator provides a description of Clarence Hervey, one of the suitors of the novel’s protagonist, Belinda Portman. Mrs. Stanhope, Belinda’s aunt, hopes to improve her niece’s social prospects and therefore has arranged to have Belinda stay with the fashionable Lady Delacour.

Read the passage carefully. Then write an essay in which you analyze Clarence Hervey’s complex character as Edgeworth develops it through such literary techniques as tone, point of view, and language.

Clarence Hervey might have been more than a

pleasant young man, if he had not been smitten with the desire of being thought superior in every thing, and of being the most admired person in all companies. He had been early flattered with the idea 5

that he was a man of genius; and he imagined that, as such, he was entitled to be imprudent, wild, and eccentric. He affected singularity, in order to establish his claims to genius. He had considerable literary talents, by which he was distinguished at Oxford; but 10

he was so dreadfully afraid of passing for a pedant, that when he came into the company of the idle and the ignorant, he pretended to disdain every species of knowledge. His chameleon character seemed to vary in different lights, and according to the different 15

situations in which he happened to be placed. He could be all things to all men—and to all women. He was supposed to be a favourite with the fair sex; and of all his various excellencies and defects, there was none on which he valued himself so much as on his 20

gallantry. He was not profligate; he had a strong sense of humour, and quick feelings of humanity; but he was so easily led, or rather so easily excited by his companions, and his companions were now of such a sort, that it was probable he would soon become 25

vicious. As to his connexion with Lady Delacour, he would have started with horror at the idea of disturbing the peace of a family; but in her family, he said, there was no peace to disturb; he was vain of having it seen by the world that he was distinguished 30

by a lady of her wit and fashion, and he did not think it incumbent on him to be more scrupulous or more

attentive to appearances than her ladyship. By Lord Delacour’s jealousy he was sometimes provoked, sometimes amused, and sometimes 35

flattered. He was constantly of all her ladyship’s parties in public and private; consequently he saw Belinda almost every day, and every day he saw her with increasing admiration of her beauty, and with increasing dread of being taken in to marry a niece 40

of ‘the catch-match-maker,’ the name by which Mrs Stanhope was known amongst the men of his acquaintance. Young ladies who have the misfortune to be conducted by these artful dames, are always supposed to be partners in all the speculations, 45

though their names may not appear in the firm. If he had not been prejudiced by the character of her aunt, Mr Hervey would have thought Belinda an undesigning, unaffected girl; but now he suspected her of artifice in every word, look, and motion; and 50

even when he felt himself most charmed by her powers of pleasing, he was most inclined to despise her, for what he thought such premature proficiency in scientific coquetry. He had not sufficient resolution to keep beyond the sphere of her attraction; but 55

frequently, when he found himself within it, he cursed his folly, and drew back with sudden terror.

Line

Page 58: 2004 AP English Literature and Composition Free …bayareatutoring.com/APlitprompts.pdf · AP® English Literature and Composition 2004 Free-Response Questions Form B ... present,

2010 AP® ENGLISH LITERATURE AND COMPOSITION FREE-RESPONSE QUESTIONS

© 2010 The College Board. Visit the College Board on the Web: www.collegeboard.com.

-4-

Question 3

(Suggested time—40 minutes. This question counts as one-third of the total essay section score.) Palestinian American literary theorist and cultural critic Edward Said has written that “Exile is strangely compelling to think about but terrible to experience. It is the unhealable rift forced between a human being and a native place, between the self and its true home: its essential sadness can never be surmounted.” Yet Said has also said that exile can become “a potent, even enriching” experience. Select a novel, play, or epic in which a character experiences such a rift and becomes cut off from “home,” whether that home is the character’s birthplace, family, homeland, or other special place. Then write an essay in which you analyze how the character’s experience with exile is both alienating and enriching, and how this experience illuminates the meaning of the work as a whole. You may choose a work from the list below or one of comparable literary merit. Do not merely summarize the plot. The American Angle of Repose Another Country As You Like It Brave New World Crime and Punishment Doctor Zhivago Heart of Darkness Invisible Man Jane Eyre Jasmine Jude the Obscure King Lear The Little Foxes Madame Bovary The Mayor of Casterbridge My Ántonia

Obasan The Odyssey One Day in the Life of Ivan Denisovich The Other Paradise Lost The Poisonwood Bible A Portrait of the Artist as a Young Man The Road Robinson Crusoe Rosencrantz and Guildenstern Are Dead Sister Carrie Sister of My Heart Snow Falling on Cedars The Tempest Things Fall Apart The Women of Brewster Place Wuthering Heights

STOP

END OF EXAM

Page 59: 2004 AP English Literature and Composition Free …bayareatutoring.com/APlitprompts.pdf · AP® English Literature and Composition 2004 Free-Response Questions Form B ... present,

AP® English Literature and Composition2002 Free-Response Questions

Form B

These materials were produced by Educational Testing Service® (ETS®), which develops and administers the examinations of the Advanced Placement Program for the College Board. The College Board and Educational Testing Service (ETS) are dedicated to the principle of equal opportunity, and their

programs, services, and employment policies are guided by that principle.

The College Board is a national nonprofit membership association dedicated to preparing, inspiring, and connecting students to college and opportunity. Founded in 1900, the association is composed of more than 4,200 schools, colleges, universities, and other educational organizations. Each year, the College Board serves over three million students and their parents, 22,000 high schools, and 3,500 colleges, through major programs and services in

college admission, guidance, assessment, financial aid, enrollment, and teaching and learning. Among its best-known programs are the SAT®, the PSAT/NMSQT®, and the Advanced Placement Program® (AP®). The College Board is committed to the principles of equity and

excellence, and that commitment is embodied in all of its programs, services, activities, and concerns.

Copyright © 2002 by College Entrance Examination Board. All rights reserved. College Board, Advanced Placement Program, AP, SAT, and the acorn logo are registered trademarks of the College Entrance Examination Board. APIEL is a trademark owned by the College Entrance Examination Board. PSAT/NMSQT is a

registered trademark jointly owned by the College Entrance Examination Board and the National Merit Scholarship Corporation. Educational Testing Service and ETS are registered trademarks of Educational Testing Service.

The materials included in these files are intended for use by AP teachers for course and exam preparation in the classroom; permission for any other use must be

sought from the Advanced Placement Program®. Teachers may reproduce them, in whole or in part, in limited quantities, for face-to-face teaching purposes but may not mass distribute the materials, electronically or otherwise. These materials and

any copies made of them may not be resold, and the copyright notices must be retained as they appear here. This permission does not apply to any third-party

copyrights contained herein.

Page 60: 2004 AP English Literature and Composition Free …bayareatutoring.com/APlitprompts.pdf · AP® English Literature and Composition 2004 Free-Response Questions Form B ... present,

2002 AP®

ENGLISH LITERATURE AND COMPOSITION FREE-RESPONSE QUESTIONS (Form B)

Copyright © 2002 by College Entrance Examination Board. All rights reserved. Advanced Placement Program and AP are registered trademarks of the College Entrance Examination Board.

GO ON TO THE NEXT PAGE. 2

ENGLISH LITERATURE AND COMPOSITION

SECTION II

Total Time—2 hours

Question 1

(Suggested time—40 minutes. This question counts for one-third of the total essay section score.)

Read carefully the following passage from the beginning of a contemporary novel. Note the author’s use of such elements as diction, syntax, imagery, and figurative language. Then write an essay in which you analyze how the author’s use of language generates a vivid impression of Quoyle as a character.

Here is an account of a few years in the life of Quoyle, born in Brooklyn and raised in a shuffle of dreary upstate towns.

Hive-spangled, gut roaring with gas and cramp, he survived childhood; at the state university, hand clapped over his chin, he camouflaged torment with smiles and silence. Stumbled through his twenties and into his thirties learning to separate his feelings from his life, counting on nothing. He ate prodigiously, liked a ham knuckle, buttered spuds.

His jobs: distributor of vending machine candy, all-night clerk in a convenience store, a third-rate newspaperman. At thirty-six, bereft, brimming with grief and thwarted love, Quoyle steered away to Newfoundland, the rock that had generated his ances-tors, a place he had never been nor thought to go. A watery place. And Quoyle feared water, could not swim. Again and again the father had broken his clenched grip and thrown him into pools, brooks, lakes, and surf. Quoyle knew the flavor of brack and waterweed.

From this youngest son’s failure to dog-paddle the father saw other failures multiply like an explosion of virulent cells—failure to speak clearly; failure to sit up straight; failure to get up in the morning; failure in atti-tude; failure in ambition and ability; indeed, in every-thing. His own failure.

Quoyle shambled, a head taller than any child around him, was soft. He knew it. “Ah, you lout,” said the father. But no pygmy himself. And brother Dick, the father’s favorite, pretended to throw up when Quoyle came into a room, hissed “Snotface, Ugly Pig, Warthog, Stupid, Stinkbomb, Greasebag,” pummeled and kicked until Quoyle curled, hands over head, sniveling, on the linoleum. All stemmed from Quoyle’s chief failure, a failure of normal appearance.

A great damp loaf of a body. At six he weighed eighty pounds. At sixteen he was buried under a casement of flesh. Head shaped like a crenshaw,

1 no

neck, reddish hair ruched2 back. Features as bunched as

kissed fingertips. Eyes the color of plastic. The mon-strous chin, a freakish shelf jutting from the lower face.

Some anomalous gene had fired up at the moment of his begetting as a single spark sometimes leaps from banked coals, had given him a giant’s chin. As a child he invented stratagems to deflect stares; a smile, down-cast gaze, the right hand darting up to cover the chin.

His earliest sense of self was as a distant figure: there in the foreground was his family; here, at the limit of the far view, was he. Until he was fourteen he cherished the idea that he had been given to the wrong family, that somewhere his real people, saddled with the changeling of the Quoyles, longed for him. Then, foraging in a box of excursion mementoes, he found photographs of his father beside brothers and sisters at a ship’s rail. A girl, somewhat apart from the others, looked toward the sea, eyes squinted, as though she could see the port of destination a thousand miles south. Quoyle recognized himself in their hair, their legs, and arms. That sly-looking lump in the shrunken sweater, his father. On the back, scribbled in blue pencil, “Leaving Home, 1946.”

At the university he took courses he couldn’t under-stand, humped back and forth without speaking to any-one, went home for weekends of excoriation. At last he dropped out of school and looked for a job, kept his hand over his chin.

Nothing was clear to lonesome Quoyle. His thoughts churned like the amorphous thing that ancient sailors, drifting into arctic half-light, called the Sea Lung; a heaving sludge of ice under fog where air blurred into water, where liquid was solid, where solids dissolved, where the sky froze and light and dark muddled. 1A crenshaw is a variety of winter melon.

2A ruche is a pleat or ruffle used for decorating garments.

Line

(5)

(10)

(15)

(20)

(25)

(30)

(35)

(40)

(45)

(50)

(55)

(60)

(65)

(70)

Page 61: 2004 AP English Literature and Composition Free …bayareatutoring.com/APlitprompts.pdf · AP® English Literature and Composition 2004 Free-Response Questions Form B ... present,

2002 AP®

ENGLISH LITERATURE AND COMPOSITION FREE-RESPONSE QUESTIONS (Form B)

Copyright © 2002 by College Entrance Examination Board. All rights reserved. Advanced Placement Program and AP are registered trademarks of the College Entrance Examination Board.

GO ON TO THE NEXT PAGE. 3

Question 2

(Suggested time—40 minutes. This question counts as one-third of the total essay section score.) The following poem is a villanelle, a form having strict rules of rhyme, meter, and repetition. Read the poem carefully. Then write a well-organized essay in which you analyze how the formal elements of the poem contribute to its meaning.

If I Could Tell You Time will say nothing but I told you so, Time only knows the price we have to pay; If I could tell you I would let you know. If we should weep when clowns put on their show, If we should stumble when musicians play, Time will say nothing but I told you so. There are no fortunes to be told, although, Because I love you more than I can say, If I could tell you I would let you know. The winds must come from somewhere when they blow, There must be reasons why the leaves decay; Time will say nothing but I told you so. Perhaps the roses really want to grow, The vision seriously intends to stay; If I could tell you I would let you know. Suppose the lions all get up and go, And all the brooks and soldiers run away; Will Time say nothing but I told you so? If I could tell you I would let you know.

From W. H. Auden: Collected Poems by W. H. Auden.

Copyright © 1945 by W. H. Auden. Reprinted by permission of

Random House, Inc.

Line

(5)

(10)

(15)

Page 62: 2004 AP English Literature and Composition Free …bayareatutoring.com/APlitprompts.pdf · AP® English Literature and Composition 2004 Free-Response Questions Form B ... present,

2002 AP®

ENGLISH LITERATURE AND COMPOSITION FREE-RESPONSE QUESTIONS (Form B)

Copyright © 2002 by College Entrance Examination Board. All rights reserved. Advanced Placement Program and AP are registered trademarks of the College Entrance Examination Board.

4

Question 3

(Suggested time—40 minutes. This question counts as one-third of the total essay section score.) Often in literature a character’s success in achieving goals depends on keeping a secret and divulging it only at the right moment, if at all. Choose a novel or play of literary merit that requires a character to keep a secret. In a well-organized essay, briefly explain the necessity for secrecy and how the character’s choice to reveal or keep the secret affects the plot and contributes to the meaning of the work as a whole. You may select a word from the list below, or you may choose another work of recognized literary merit suitable to the topic. Do NOT write about a short story, poem or film. Beloved Cat on a Hot Tin Roof Crime and Punishment Death of a Salesman A Doll House Ghosts Great Expectations The Great Gatsby Heart of Darkness The Importance of Being Earnest

Jane Eyre Light in August Macbeth The Mayor of Casterbridge The Piano Lesson The Playboy of the Western World Romeo and Juliet The Scarlet Letter Song of Solomon

END OF EXAMINATION

Page 63: 2004 AP English Literature and Composition Free …bayareatutoring.com/APlitprompts.pdf · AP® English Literature and Composition 2004 Free-Response Questions Form B ... present,

AP® English Literature and Composition2003 Free-Response Questions

Form B

These materials were produced by Educational Testing Service® (ETS®), which develops and administers the examinations of the Advanced Placement Program for the College Board. The College Board and Educational Testing Service (ETS) are dedicated to the principle of equal opportunity, and their

programs, services, and employment policies are guided by that principle.

The College Board is a national nonprofit membership association whose mission is to prepare, inspire, and connect students to college and opportunity. Founded in 1900, the association is composed of more than 4,300 schools, colleges, universities, and other educational organizations. Each year, the College Board serves over three million students and their parents, 22,000 high schools, and 3,500 colleges through major programs and services in college admissions, guidance, assessment, financial aid, enrollment, and teaching and learning. Among its best-known programs are the SAT®, the

PSAT/NMSQT®, and the Advanced Placement Program® (AP®). The College Board is committed to the principles of equity and excellence, and that commitment is embodied in all of its programs, services, activities, and concerns.

For further information, visit www.collegeboard.com

Copyright © 2003 College Entrance Examination Board. All rights reserved. College Board, Advanced Placement Program, AP, AP Vertical Teams, APCD, Pacesetter, Pre-AP, SAT, Student Search Service, and the acorn logo are registered trademarks of the College Entrance Examination Board.

AP Central is a trademark owned by the College Entrance Examination Board. PSAT/NMSQT is a registered trademark jointly owned by the College Entrance Examination Board and the National Merit Scholarship Corporation. Educational Testing Service and ETS are registered trademarks of

Educational Testing Service. Other products and services may be trademarks of their respective owners.

For the College Board’s online home for AP professionals, visit AP Central at apcentral.collegeboard.com.

The materials included in these files are intended for use by AP teachers for course and exam preparation; permission for any other use must be

sought from the Advanced Placement Program®. Teachers may reproduce them, in whole or in part, in limited quantities for noncommercial, face-to-face teaching

purposes. This permission does not apply to any third-party copyrights contained herein. This material may not be mass distributed, electronically or otherwise.

These materials and any copies made of them may not be resold, and the copyright notices must be retained as they appear here.

Page 64: 2004 AP English Literature and Composition Free …bayareatutoring.com/APlitprompts.pdf · AP® English Literature and Composition 2004 Free-Response Questions Form B ... present,

2003 AP® ENGLISH LITERATURE AND COMPOSITION FREE-RESPONSE QUESTIONS (Form B)

Copyright © 2003 by College Entrance Examination Board. All rights reserved. Available to AP professionals at apcentral.collegeboard.com and to

students and parents at www.collegeboard.com/apstudents.

GO ON TO THE NEXT PAGE. 2

ENGLISH LITERATURE AND COMPOSITION SECTION II

Total time—2 hours

Question 1

(Suggested time—40 minutes. This question counts as one-third of the total essay section score.)

The following poem is taken from Modern Love, a poetic sequence by the English writer George Meredith. Read the poem carefully. Then write a well-organized essay in which you analyze how the poet conveys a view of “modern love.”

By this he knew she wept with waking eyes: That, at his hand’s light quiver by her head, The strange low sobs that shook their common bed Were called into her with a sharp surprise, And strangled mute, like little gaping snakes, 5

Dreadfully venomous to him. She lay Stone-still, and the long darkness flowed away With muffled pulses. Then, as midnight makes Her giant heart of Memory and Tears Drink the pale drug of silence, and so beat 10

Sleep’s heavy measure, they from head to feet Were moveless, looking through their dead black years, By vain regret scrawled over the blank wall. Like sculptured effigies* they might be seen Upon their marriage-tomb, the sword between; 15

Each wishing for the sword that severs all. (1862) * The stone figures of a husband and wife carved on medieval tombs

Line

Page 65: 2004 AP English Literature and Composition Free …bayareatutoring.com/APlitprompts.pdf · AP® English Literature and Composition 2004 Free-Response Questions Form B ... present,

2003 AP® ENGLISH LITERATURE AND COMPOSITION FREE-RESPONSE QUESTIONS (Form B)

Copyright © 2003 by College Entrance Examination Board. All rights reserved. Available to AP professionals at apcentral.collegeboard.com and to

students and parents at www.collegeboard.com/apstudents.

GO ON TO THE NEXT PAGE. 3

Question 2

(Suggested time — 40 minutes. This question counts as one-third of the total essay section score.) Read the following passage from Joyce Carol Oates’s novel We Were the Mulvaneys (1996). Then, in a well-organized essay, analyze the literary techniques Oates uses to characterize the speaker, Judd Mulvaney. Support your analysis with specific references to the passage.

That time in our lower driveway, by the brook. I was straddling my bike staring down into the water. Fast-flowing clear water, shallow, shale beneath, and lots of leaves. Sky the color of lead and the light mostly drained so I couldn’t see my face only the 5

dark shape of a head that could be anybody’s head. Hypnotizing myself the way kids do. Lonely kids, or kids not realizing they’re lonely. The brook was flowing below left to right (east to west, though at a slant) and I stood immobile leaning on the railing 10

(pretty damn rotted: I’d tell Dad it needed to be replaced with new planks, we could do it together) until it began to happen as it always does the water gets slower and slower and you’re the one who begins to move — oh boy! we-ird! scary and ticklish 15

in the groin and I leaned farther and farther over the rail staring into the water and I was moving, moving helplessly forward, it seemed I was moving somehow upward, rising into the air, helpless, in that instant aware of my heart beating ONEtwothree 20

ONEtwothree! thinking Every heartbeat is past and gone! Every heartbeat is past and gone! A chill came over me, I began to shiver. It wasn’t warm weather now but might have been late as November, most of the leaves blown from the trees. Only the evergreens 25

and some of the black birches remaining but it’s a fact when dry yellow leaves (like on the birches) don’t fall from a tree the tree is partly dead. A light gritty film of snow on the ground, darkest in the crevices where you’d expect shadow so it was like a film negative. 30

Every heartbeat is past and gone! Every heartbeat is past and gone! in a trance that was like a trance of fury, raging hurt Am I going to die? because I did not believe that Judd Mulvaney could die. (Though on a

farm living things are dying, dying, dying all the time, 35

and many have been named, and others are born taking their places not even knowing that they are taking the places of those who have died.) So I knew, I wasn’t a dope, but I didn’t know — not really. Aged eleven, or maybe twelve. Leaning over the rotted rail 40

gaping at the water hypnotized and scared and suddenly there came Dad and Mike in the mud-colored Ford pickup (Might as well buy our vehicles mud-colored to begin with, saves time, was Dad’s logic) barreling up the drive, bouncing and rattling. 45

On the truck’s doors were neat curving white letters sweet to see MULVANEY ROOFING (716) 689-8329. They’d be passing so close my bike might snag in a fender so I grabbed it and hauled it to the side. Mike had rolled down his window to lean out and pretend 50

to cuff at my head—“Hey Ranger-kid: what’s up?” Dad at the wheel grinned and laughed and next second they were past, the pickup in full throttle ascending the drive. And I looked after them, these two people so remarkable to me, my dad who was like 55

nobody else’s dad and my big brother who was —well, Mike Mulvaney: “Mule” Mulvaney —and the most terrible thought came to me.

Them, too. All of them. Every heartbeat past and gone. 60

It stayed with me for a long time, maybe forever. Not just that I would lose the people I loved, but they would lose me — Judson Andrew Mulvaney. And they knew nothing of it. (Did they?) And I, just a skinny kid, the runt of the litter at High Point Farm, would 65

have to pretend not to know what I knew.

Line

Page 66: 2004 AP English Literature and Composition Free …bayareatutoring.com/APlitprompts.pdf · AP® English Literature and Composition 2004 Free-Response Questions Form B ... present,

2003 AP® ENGLISH LITERATURE AND COMPOSITION FREE-RESPONSE QUESTIONS (Form B)

Copyright © 2003 by College Entrance Examination Board. All rights reserved. Available to AP professionals at apcentral.collegeboard.com and to

students and parents at www.collegeboard.com/apstudents.

4

Question 3

(Suggested time—40 minutes. This question counts as one-third of the total essay section score.) Novels and plays often depict characters caught between colliding cultures—national, regional, ethnic, religious, institutional. Such collisions can call a character’s sense of identity into question. Select a novel or play in which a character responds to such a cultural collision. Then write a well-organized essay in which you describe the character’s response and explain its relevance to the work as a whole. You may select a work from the list below or choose another appropriate novel or play of similar literary merit. Avoid mere plot summary.

The Age of Innocence A Bend in the River Bone Catch-22 Ceremony Daisy Miller Dreaming In Cuban Dutchman A Fine Balance The Grapes of Wrath Great Expectations The Handmaid’s Tale Heart of Darkness Invisible Man The Joy Luck Club

Mansfield Park “Master Harold” . . . and the boys The Merchant of Venice Monkey Bridge My Ántonia My Name is Asher Lev Native Speaker Othello The Portrait of a Lady Pygmalion The Remains of the Day A Room With a View The Tempest Things Fall Apart Typical American

END OF EXAMINATION

Page 67: 2004 AP English Literature and Composition Free …bayareatutoring.com/APlitprompts.pdf · AP® English Literature and Composition 2004 Free-Response Questions Form B ... present,

AP English Literature and Composition 2001 Free-Response Questions

These materials were produced by Educational Testing Service (ETS), which develops and administers the examinations of the Advanced Placement Program for the College Board. The College Board and Educational Testing Service (ETS) are dedicated to the principle of equal opportunity, and their

programs, services, and employment policies are guided by that principle.

The College Board is a national nonprofit membership association dedicated to preparing, inspiring, and connecting students to college and opportunity. Founded in 1900, the association is composed of more than 3,900 schools, colleges, universities, and other educational organizations. Each year, the College Board serves over three million students and their parents, 22,000 high schools, and 3,500 colleges, through major programs and services in

college admission, guidance, assessment, financial aid, enrollment, and teaching and learning. Among its best-known programs are the SAT®, the PSAT/NMSQT�, the Advanced Placement Program® (AP®), and Pacesetter®. The College Board is committed to the principles of equity and

excellence, and that commitment is embodied in all of its programs, services, activities, and concerns.

Copyright © 2001 by College Entrance Examination Board. All rights reserved. College Board, Advanced Placement Program, AP, and the acorn logo are registered trademarks of the College Entrance Examination Board.

The materials included in these files are intended for use by AP teachers for course and exam preparation in the classroom; permission for any other use must be

sought from the Advanced Placement Program. Teachers may reproduce them, in whole or in part, in limited quantities, for face-to-face teaching purposes but may not mass distribute the materials, electronically or otherwise. These materials and

any copies made of them may not be resold, and the copyright notices must be retained as they appear here. This permission does not apply to any third-party

copyrights contained herein.

Page 68: 2004 AP English Literature and Composition Free …bayareatutoring.com/APlitprompts.pdf · AP® English Literature and Composition 2004 Free-Response Questions Form B ... present,

2001 AP® ENGLISH LITERATURE AND COMPOSITIONFREE-RESPONSE QUESTIONS

Copyright © 2001 by College Entrance Examination Board. All rights reserved.Advanced Placement Program and AP are registered trademarks of the College Entrance Examination Board.

GO ON TO THE NEXT PAGE.2

ENGLISH LITERATURE AND COMPOSITIONSECTION II

Total time—2 hours

Question 1

(Suggested time—40 minutes. This question counts as one-third of the total essay section score.)

In each of the following poems, the speaker responds to the conditions of a particular place and time—England in1802 in the first poem, the United States about 100 years later in the second. Read each poem carefully. Then writean essay in which you compare and contrast the two poems and analyze the relationship between them.

London, 1802

Milton!1 thou shouldst be living at this hour:England hath need of thee: she is a fen2

Of stagnant waters: altar, sword, and pen,Fireside, the heroic wealth of hall and bower,Have forfeited their ancient English dower35

Of inward happiness. We are selfish men;Oh! raise us up, return to us again;And give us manners, virtue, freedom, power.Thy soul was like a Star, and dwelt apart;Thou hadst a voice whose sound was like the sea:10

Pure as the naked heavens, majestic, free,So didst thou travel on life’s common way,In cheerful godliness; and yet thy heartThe lowliest duties on herself did lay.

—William Wordsworth (1770-1850)

1 John Milton (1608-1674), English poet and political writer, author ofParadise Lost, whose famous essay against censorship, “Areopagitica,”championed the cause of liberty and public virtue.

2 Swamp3 Natural endowment

Douglass*

Ah, Douglass, we have fall’n on evil days,Such days as thou, not even thou didst know,When thee, the eyes of that harsh long ago

Saw, salient, at the cross of devious ways,And all the country heard thee with amaze.5

Not ended then, the passionate ebb and flow,The awful tide that battled to and fro;

We ride amid a tempest of dispraise.

Now, when the waves of swift dissension swarm,And Honor, the strong pilot, lieth stark,10

Oh for thy voice high-sounding o’er the storm,For thy strong arm to guide the shivering bark,

The blast-defying power of thy form,To give us comfort through the lonely dark.

Paul Laurence Dunbar (1872-1906)

*Frederick Douglass (1817-1895), American writer, former slave, whoseautobiography (1845) made him a leader in the abolitionist cause.

Line Line

Page 69: 2004 AP English Literature and Composition Free …bayareatutoring.com/APlitprompts.pdf · AP® English Literature and Composition 2004 Free-Response Questions Form B ... present,

2001 AP® ENGLISH LITERATURE AND COMPOSITIONFREE-RESPONSE QUESTIONS

Copyright © 2001 by College Entrance Examination Board. All rights reserved.Advanced Placement Program and AP are registered trademarks of the College Entrance Examination Board.

GO ON TO THE NEXT PAGE.3

Question 2

(Suggested time — 40 minutes. This question counts as one-third of the total essay section score.)

The passage below is taken from the novel Tom Jones (1749) by the English novelist and playwright Henry Fielding.In this scene, which occurs early in the novel, Squire Allworthy discovers an infant in his bed. Read the passagecarefully. Then, in a well-organized essay, analyze the techniques that Fielding employs in this scene to characterizeMr. Allworthy and Mrs. Deborah Wilkins.

Mr. Allworthy came to his house very late in theevening, and after a short supper with his sister,retired much fatigued to his chamber. Here, havingspent some minutes on his knees—a custom which henever broke through on any account—he was5

preparing to step into bed, when, upon opening theclothes, to his great surprise he beheld an infant,wrapt up in some coarse linen, in a sweet andprofound sleep, between his sheets. He stood sometime lost in astonishment at this sight; but, as good-10

nature had always the ascendant in his mind, he soonbegan to be touched with sentiments of compassionfor the little wretch before him. He then rang his bell,and ordered an elderly woman-servant to riseimmediately, and come to him; and in the mean time15

was so eager in contemplating the beauty ofinnocence, appearing in those lively colours withwhich infancy and sleep always display it, that histhoughts were too much engaged to reflect that he wasin his shirt when the matron came in. She had, indeed,20

given her master sufficient time to dress himself; forout of respect to him, and regard to decency, she hadspent many minutes in adjusting her hair at thelooking-glass, notwithstanding all the hurry in whichshe had been summoned by the servant, and though25

her master, for aught she knew, lay expiring in anapoplexy, or in some other fit.

It will not be wondered at that a creature who hadso strict a regard to decency in her own person shouldbe shocked at the least deviation from it in another.30

She therefore no sooner opened the door, and saw hermaster standing by the bedside in his shirt, with acandle in his hand, than she started back in a mostterrible fright, and might perhaps have swooned away,had he not now recollected his being undressed, and35

put an end to her terrors by desiring her to staywithout the door till he had thrown some clothes overhis back, and was become incapable of shocking thepure eyes of Mrs. Deborah Wilkins, who, though inthe fifty-second year of her age, vowed she had never40

beheld a man without his coat . . .When Mrs. Deborah returned into the room, and

was acquainted by her master with the finding thelittle infant, her consternation was rather greater thanhis had been; nor could she refrain from crying out,45

with great horror of accent as well as look, “My goodsir! what’s to be done?” Mr. Allworthy answered, shemust take care of the child that evening, and in themorning he would give orders to provide it a nurse.“Yes, sir,” says she; “and I hope your worship will50

send out your warrant to take up the hussy its mother,for she must be one of the neighbourhood; and Ishould be glad to see her committed to Bridewell, andwhipt at the cart’s tail. . . . but for my own part, itgoes against me to touch these misbegotten wretches,55

whom I don’t look upon as my fellow-creature.Faugh! how it stinks!. . . If I might be so bold to givemy advice, I would have it put in a basket, and sentout and laid at the churchwarden’s door. It is a goodnight, only a little rainy and windy; and if it was well60

wrapt up, and put in a warm basket, it is two to onebut it lives till it is found in the morning. But if itshould not, we have discharged our duty in takingproper care of it; and it is, perhaps, better for suchcreatures to die in a state of innocence, than to grow65

up and imitate their mothers; for nothing better can beexpected of them.”

There were some strokes in this speech which,perhaps, would have offended Mr. Allworthy had hestrictly attended to it; but he had now got one of his70

fingers into the infant’s hand, which, by its gentlepressure, seeming to implore his assistance, hadcertainly outpleaded the eloquence of Mrs. Deborah,had it been ten times greater than it was. He now gaveMrs. Deborah positive orders to take the child to her75

own bed, and to call up a maid-servant to provide itpap, and other things, against it waked. . .

Such was the discernment of Mrs. Wilkins, andsuch the respect she bore her master, under whom sheenjoyed a most excellent place, that her scruples gave80

way to his peremptory commands; and she took thechild under her arms, without any apparent disgust atthe illegality of its birth; and declaring it was a sweetlittle infant, walked off with it to her own chamber.

Line

Page 70: 2004 AP English Literature and Composition Free …bayareatutoring.com/APlitprompts.pdf · AP® English Literature and Composition 2004 Free-Response Questions Form B ... present,

2001 AP® ENGLISH LITERATURE AND COMPOSITIONFREE-RESPONSE QUESTIONS

Copyright © 2001 by College Entrance Examination Board. All rights reserved.Advanced Placement Program and AP are registered trademarks of the College Entrance Examination Board.

4

Question 3

(Suggested time — 40 minutes. This question counts as one-third of the total essay section score.)

One definition of madness is “mental delusion or the eccentric behavior arising from it.” But Emily Dickinson wrote

Much madness is divinest Sense—To a discerning Eye—

Novelists and playwrights have often seen madness with a “discerning Eye.” Select a novel or play in which acharacter’s apparent madness or irrational behavior plays an important role. Then write a well-organized essayin which you explain what this delusion or eccentric behavior consists of and how it might be judged reasonable.Explain the significance of the “madness” to the work as a whole. Do not merely summarize the plot.

You may select a work from the list below or choose another novel or play of literary merit.

As I Lay DyingBelovedCatch-22The Catcher in the RyeCeremonyComing Through SlaughterCrime and PunishmentDancing at LughnasaDon QuixoteAn Enemy of the PeopleEquusThe FatherGoing After CacciatoGreat ExpectationsGulliver’s Travels

Heart of DarknessInvisible ManKing LearMedeaMoby-DickNative SonOf Mice and MenOne Flew Over the Cuckoo’s NestPale FireThe Sound and the FuryA Streetcar Named DesireWaiting for GodotWuthering HeightsThe Zoo Story

END OF EXAMINATION

Page 71: 2004 AP English Literature and Composition Free …bayareatutoring.com/APlitprompts.pdf · AP® English Literature and Composition 2004 Free-Response Questions Form B ... present,

AP® English Literature and Composition 2002 Free-Response Questions

These materials were produced by Educational Testing Service® (ETS®), which develops and administers the examinations of the Advanced Placement Program for the College Board. The College Board and Educational Testing Service (ETS) are dedicated to the principle of equal opportunity, and their

programs, services, and employment policies are guided by that principle.

The College Board is a national nonprofit membership association dedicated to preparing, inspiring, and connecting students to college and opportunity. Founded in 1900, the association is composed of more than 4,200 schools, colleges, universities, and other educational organizations. Each year, the College Board serves over three million students and their parents, 22,000 high schools, and 3,500 colleges, through major programs and services in

college admission, guidance, assessment, financial aid, enrollment, and teaching and learning. Among its best-known programs are the SAT®, the PSAT/NMSQT®, and the Advanced Placement Program® (AP®). The College Board is committed to the principles of equity and

excellence, and that commitment is embodied in all of its programs, services, activities, and concerns.

Copyright © 2002 by College Entrance Examination Board. All rights reserved. College Board, Advanced Placement Program, AP, SAT, and the acorn logo are registered trademarks of the College Entrance Examination Board. APIEL is a trademark owned by the College Entrance Examination Board. PSAT/NMSQT is a

registered trademark jointly owned by the College Entrance Examination Board and the National Merit Scholarship Corporation. Educational Testing Service and ETS are registered trademarks of Educational Testing Service.

The materials included in these files are intended for use by AP teachers for course and exam preparation in the classroom; permission for any other use must be

sought from the Advanced Placement Program®. Teachers may reproduce them, in whole or in part, in limited quantities, for face-to-face teaching purposes but may not mass distribute the materials, electronically or otherwise. These materials and

any copies made of them may not be resold, and the copyright notices must be retained as they appear here. This permission does not apply to any third-party

copyrights contained herein.

Page 72: 2004 AP English Literature and Composition Free …bayareatutoring.com/APlitprompts.pdf · AP® English Literature and Composition 2004 Free-Response Questions Form B ... present,

2002 AP® ENGLISH LITERATURE AND COMPOSITION FREE-RESPONSE QUESTIONS

Copyright © 2002 by College Entrance Examination Board. All rights reserved. Advanced Placement Program and AP are registered trademarks of the College Entrance Examination Board.

GO ON TO THE NEXT PAGE. 2

ENGLISH LITERATURE AND COMPOSITION SECTION II

Total time—2 hours

Question 1

(Suggested time— 40 minutes. This question counts as one-third of the total essay section score.)

In the following excerpt from a recent British novel, the narrator, a young man in his early twenties, is attending a play with his new girlfriend Isabel when she unexpectedly discovers that her parents are in the theater. Read the passage carefully. Then write an essay in which you analyze how the author produces a comic effect.

Oh my God, I think that’s my mum over there,’ she gasped.

‘Where?’ ‘By the pillar. Careful, don’t look. What is she

doing here? And what’s that dress? It looks like a 5

willow tree. Where’s Dad? I hope she didn’t come with one of her gentlemen friends. She’s really too old for that.’

‘Did you tell her you were going?’ ‘No, I mean, I said I wanted to see the play, but I 10

didn’t let on I had tickets for tonight.’ ‘She’s talking to someone. Can you see?’ ‘Phew, it’s my dad. He must have gone off to buy

programmes. And he’s about to sneeze. Look, there we go, aaahhtchooo. Out comes his red handkerchief. 15

I just hope they don’t spot us and we can escape quickly at the end. With any luck, they’ll be too busy arguing to glance up here. This is prime argument territory for them, Mum will be asking Dad where he put the car park ticket and he’ll get flustered because 20

he’ll just have dropped it into a bin by mistake.’ Luck was not on Isabel’s side, for a moment later,

Christopher Rogers happened to glance up to the gallery and recognized his eldest daughter, in the midst of trying her best not to recognize him. So that 25

she might cease to dwell in ignorance, Christopher stood up in the middle of the elegantly suited and scented audience, and began making the vigorous hand gestures of a man waving off a departing cruise ship. In case Isabel had not spotted this maniac, her 30

mother was in turn informed of her eldest daughter’s location, and decided that the presence of four hundred people in the auditorium should be no impediment to her desire to shout ‘Isabel’ at top pitch and with all the excitement of a woman recognizing a 35

long-lost friend on the deck of an in-coming cruise ship.

Isabel smiled feebly, turned a beetroot shade and repeated in panicked diction, ‘I can’t believe this,

please let them shut up.’ 40

Not a second too soon, Lorca* came to the rescue, the lights faded, and Mr. and Mrs. Rogers reluctantly took their seats, pointing ominously to an exit sign by way of interval rendezvous.

An hour and a quarter of Spanish domestic drama 45

later, we found ourselves at the bar. ‘What are you doing here, Mum?’ asked Isabel. ‘Why shouldn’t I be here? You’re not the only one

who does fancy things with your evenings. Your father and I have a right to go out once in a while.’ 50

‘I’m sure, I didn’t mean it like that, it’s just I’m surprised at the coincidence.’

‘Where did you buy this dress? Is that the one I paid for at Christmas?’

‘No, Mum, I got it myself last week.’ 55

‘Oh, well, it’s very nice, pity you don’t have more of a cleavage for it, but that’s your father’s fault. You know what all the women in his family are like.’

‘How are you Dad?’ Isabel turned to ask her father, who was looking up at the ceiling with an intent 60

expression. ‘Dad?’ repeated Isabel. ‘Yes, darling, how are you, my bean? Enjoying the

show?’ ‘Yup, and you? What are you staring at up there?’ 65

‘I’m looking at the light fixtures they have. They’re new tungsten bulbs, Japanese things, quite wonderful, they use only a small amount of electricity but give off a very nice light.’

‘Oh, great, Dad. And, ehm, there’s someone I’d like 70

you to both meet.’ ‘Delighted,’ said Mrs. Rogers, confiding in me

almost at once: ‘She’s a lovely girl really,’ in case my theatre companion had inspired doubts to the contrary. 75

‘Thanks, Mum,’ said Isabel wearily, as though the statement were no one-off.

‘Don’t mind her, bean, she’s had a hard day,’

Line

Page 73: 2004 AP English Literature and Composition Free …bayareatutoring.com/APlitprompts.pdf · AP® English Literature and Composition 2004 Free-Response Questions Form B ... present,

2002 AP® ENGLISH LITERATURE AND COMPOSITION FREE-RESPONSE QUESTIONS

Copyright © 2002 by College Entrance Examination Board. All rights reserved. Advanced Placement Program and AP are registered trademarks of the College Entrance Examination Board.

GO ON TO THE NEXT PAGE. 3

explained Dad, now looking more horizontally at the world. 80

‘My day would be fine if I wasn’t lumbered with someone who kept losing tickets to the car park,’ snapped Mrs. Rogers.

‘Dad! You haven’t?’ ‘Yes, I’m afraid I have. They’re so fiddly these days, 85

they fall right out of one’s hands.’

—Alain de Botton, Kiss and Tell

* Federico Garcia Lorca (1898-1936): Spanish poet and playwright

Page 74: 2004 AP English Literature and Composition Free …bayareatutoring.com/APlitprompts.pdf · AP® English Literature and Composition 2004 Free-Response Questions Form B ... present,

2002 AP® ENGLISH LITERATURE AND COMPOSITION FREE-RESPONSE QUESTIONS

Copyright © 2002 by College Entrance Examination Board. All rights reserved. Advanced Placement Program and AP are registered trademarks of the College Entrance Examination Board.

GO ON TO THE NEXT PAGE. 4

Question 2

(Suggested time—40 minutes. This question counts as one-third of the total essay section score.) Read the following poem carefully. Then, taking into consideration the title of the poem, analyze how the poetic devices convey the speaker’s attitude toward the sinking of the ship.

The Convergence of the Twain

(Lines on the loss of the Titanic

1)

I

In a solitude of the sea Deep from human vanity, And the Pride of Life that planned her, stilly couches she.

II Steel chambers, late the pyres Of her salamandrine2 fires, 5

Cold currents thrid,3 and turn to rhythmic tidal lyres.

III Over the mirrors meant To glass the opulent The sea-worm crawls—grotesque, slimed, dumb, indifferent.

IV Jewels in joy designed 10

To ravish the sensuous mind Lie lightless, all their sparkles bleared and black and blind.

V Dim moon-eyed fishes near Gaze at the gilded gear

And query: “What does this vaingloriousness down here?”. . . 15

VI

Well: while was fashioning This creature of cleaving wing,

The Immanent Will that stirs and urges everything

VII Prepared a sinister mate For her—so gaily great— 20

A Shape of Ice, for the time far and dissociate.

VIII And as the smart ship grew In stature, grace and hue,

In shadowy silent distance grew the Iceberg too.

Line

Page 75: 2004 AP English Literature and Composition Free …bayareatutoring.com/APlitprompts.pdf · AP® English Literature and Composition 2004 Free-Response Questions Form B ... present,

2002 AP® ENGLISH LITERATURE AND COMPOSITION FREE-RESPONSE QUESTIONS

Copyright © 2002 by College Entrance Examination Board. All rights reserved. Advanced Placement Program and AP are registered trademarks of the College Entrance Examination Board.

GO ON TO THE NEXT PAGE. 5

IX

Alien they seemed to be: 25

No mortal eye could see The intimate welding of their later history,

X Or sign that they were bent By paths coincident

On being anon twin halves of one august event, 30

XI

Till the Spinner of the Years Said “Now!” And each one hears,

And consummation comes, and jars two hemispheres. —Thomas Hardy

1 On the night of April 14, 1912, the British White Star liner Titanic, the largest ship afloat, collided with an iceberg and sank on her maiden voyage from Southampton to New York. Fifteen hundred of the 2,206 passengers lost their lives.

2 Bright red. The salamander was supposed to be able to live in the midst of fire.

3 Thread

Page 76: 2004 AP English Literature and Composition Free …bayareatutoring.com/APlitprompts.pdf · AP® English Literature and Composition 2004 Free-Response Questions Form B ... present,

2002 AP® ENGLISH LITERATURE AND COMPOSITION FREE-RESPONSE QUESTIONS

Copyright © 2002 by College Entrance Examination Board. All rights reserved. Advanced Placement Program and AP are registered trademarks of the College Entrance Examination Board.

6

Question 3

(Suggested time—40 minutes. This question counts as one-third of the total essay section score.)

Morally ambiguous characters—characters whose behavior discourages readers from identifying them as purely evil or purely good—are at the heart of many works of literature. Choose a novel or play in which a morally ambiguous character plays a pivotal role. Then write an essay in which you explain how the character can be viewed as morally ambiguous and why his or her moral ambiguity is significant to the work as a whole. Avoid mere plot summary. Choose a work from the list below or another novel or play of comparable literary merit.

The Age of Innocence All the King’s Men Anna Karenina The Autobiography of An Ex-Colored Man The Awakening Billy Budd Crime and Punishment Faust Fences The Glass Menagerie Great Expectations The Great Gatsby Heart of Darkness Hedda Gabler

Henry V The Mayor of Casterbridge The Merchant of Venice Mrs. Warren’s Profession Père Goriot The Picture of Dorian Gray The Plague Poccho The Scarlet Letter Silas Marner Sister Carrie Sula The Turn of the Screw Typical American

END OF EXAMINATION

Page 77: 2004 AP English Literature and Composition Free …bayareatutoring.com/APlitprompts.pdf · AP® English Literature and Composition 2004 Free-Response Questions Form B ... present,

AP® English Literature and Composition2003 Free-Response Questions

These materials were produced by Educational Testing Service® (ETS®), which develops and administers the examinations of the Advanced Placement Program for the College Board. The College Board and Educational Testing Service (ETS) are dedicated to the principle of equal opportunity, and their

programs, services, and employment policies are guided by that principle.

The College Board is a national nonprofit membership association whose mission is to prepare, inspire, and connect students to college and opportunity. Founded in 1900, the association is composed of more than 4,300 schools, colleges, universities, and other educational organizations. Each year, the

College Board serves over three million students and their parents, 22,000 high schools, and 3,500 colleges through major programs and services in college admissions, guidance, assessment, financial aid, enrollment, and teaching and learning. Among its best-known programs are the SAT®, the

PSAT/NMSQT®, and the Advanced Placement Program® (AP®). The College Board is committed to the principles of equity and excellence, and that commitment is embodied in all of its programs, services, activities, and concerns.

For further information, visit www.collegeboard.com

Copyright © 2003 College Entrance Examination Board. All rights reserved. College Board, Advanced Placement Program, AP, AP Vertical Teams, APCD, Pacesetter, Pre-AP, SAT, Student Search Service, and the acorn logo are registered trademarks of the College Entrance Examination Board.

AP Central is a trademark owned by the College Entrance Examination Board. PSAT/NMSQT is a registered trademark jointly owned by the College Entrance Examination Board and the National Merit Scholarship Corporation. Educational Testing Service and ETS are registered trademarks of

Educational Testing Service. Other products and services may be trademarks of their respective owners.

For the College Board’s online home for AP professionals, visit AP Central at apcentral.collegeboard.com.

The materials included in these files are intended for use by AP teachers

for course and exam preparation; permission for any other use must be

sought from the Advanced Placement Program®. Teachers may reproduce them, in

whole or in part, in limited quantities for noncommercial, face-to-face teaching

purposes. This permission does not apply to any third-party copyrights contained

herein. This material may not be mass distributed, electronically or otherwise.

These materials and any copies made of them may not be resold, and the

copyright notices must be retained as they appear here.

Page 78: 2004 AP English Literature and Composition Free …bayareatutoring.com/APlitprompts.pdf · AP® English Literature and Composition 2004 Free-Response Questions Form B ... present,

2003 AP® ENGLISH LITERATURE AND COMPOSITION FREE-RESPONSE QUESTIONS

Copyright © 2003 by College Entrance Examination Board. All rights reserved. Available to AP professionals at apcentral.collegeboard.com and to

students and parents at www.collegeboard.com/apstudents. GO ON TO THE NEXT PAGE.

2

ENGLISH LITERATURE AND COMPOSITION SECTION II

Total time—2 hours

Question 1

(Suggested time— 40 minutes. This question counts as one-third of the total essay section score.) The following poems are both concerned with Eros, the god of love in Greek mythology. Read the poems carefully. Then write an essay in which you compare and contrast the two concepts of Eros and analyze the techniques used to create them. �P��1

Why hast thou nothing in thy face? Thou idol of the human race, Thou tyrant of the human heart, The flower of lovely youth that art; Yea, and that standest in thy youth 5

An image of eternal Truth, With thy exuberant flesh so fair, That only Pheidias2 might compare, Ere from his chaste marmoreal3 form Time had decayed the colours warm; 10

Like to his gods in thy proud dress, Thy starry sheen of nakedness. Surely thy body is thy mind, For in thy face is nought to find, Only thy soft unchristen’d smile, 15

That shadows neither love nor guile, But shameless will and power immense, In secret sensuous innocence. O king of joy, what is thy thought? I dream thou knowest it is nought, 20

And wouldst in darkness come, but thou Makest the light where’er thou go. Ah yet no victim of thy grace, None who e’er long’d for thy embrace, Hath cared to look upon thy face. 25

—Robert Bridges (1899) 1 Eros (in Greek) 2 Greek sculptor of the fifth century B.C. 3 marble

Eros I call for love But help me, who arrives? This thug with broken nose And squinty eyes. ‘Eros, my bully boy, 5

Can this be you, With boxer lips And patchy wings askew?’ ‘Madam,’ cries Eros, ‘Know the brute you see 10

Is what long overuse Has made of me. My face that so offends you Is the sum Of blows your lust delivered 15

One by one. We slaves who are immortal Gloss your fate And are the archetypes That you create. 20

Better my battered visage, Bruised but hot, Than love dissolved in loss Or left to rot.’ —Anne Stevenson (1990)

Anne Stevenson, Collected Poems 1955-1995, Bloodaxe Books, 2000.

Line Line

Page 79: 2004 AP English Literature and Composition Free …bayareatutoring.com/APlitprompts.pdf · AP® English Literature and Composition 2004 Free-Response Questions Form B ... present,

2003 AP® ENGLISH LITERATURE AND COMPOSITION FREE-RESPONSE QUESTIONS

Copyright © 2003 by College Entrance Examination Board. All rights reserved. Available to AP professionals at apcentral.collegeboard.com and to

students and parents at www.collegeboard.com/apstudents. GO ON TO THE NEXT PAGE.

3

Question 2

(Suggested time— 40 minutes. This question counts as one-third of the total essay section score.)

The following passage is an excerpt from “The Other Paris,” a short story by the Canadian writer Mavis Gallant. Read the passage carefully. Then, in a well-written essay, explain how the author uses narrative voice and characterization to provide social commentary.

If anyone had asked Carol at what precise moment she fell in love, or where Howard Mitchell proposed to her, she would have imagined, quite sincerely, a scene that involved all at once the Seine, moonlight, barrows of violets, acacias in flower, and a confused, 5

misty background of the Eiffel tower and little crooked streets. This was what everyone expected, and she had nearly come to believe it herself.

Actually, he had proposed at lunch, over a tuna-fish salad. He and Carol had known each other less than 10

three weeks, and their conversation, until then, had been limited to their office—an American govern-ment agency—and the people in it. Carol was twenty-two; no one had proposed to her before, except an unsuitable medical student with no money and eight 15

years’ training still to go. She was under the illusion that in a short time she would be so old no one would ask her again. She accepted at once, and Howard celebrated by ordering an extra bottle of wine. Both would have liked champagne, as a more emphatic 20

symbol of the unusual, but each was too diffident to suggest it.

The fact that Carol was not in love with Howard Mitchell did not dismay her in the least. From a series of helpful college lectures on marriage she had 25

learned that a common interest, such as a liking for Irish setters, was the true basis for happiness, and that the illusion of love was a blight imposed by the film industry, and almost entirely responsible for the high rate of divorce. Similar economic backgrounds, 30

financial security, belonging to the same church—these were the pillars of the married union. By an astonishing coincidence, the fathers of Carol and Howard were both attorneys and both had been defeated in their one attempt to get elected a judge. 35

Carol and Howard were both vaguely Protestant, although a serious discussion of religious beliefs would have gravely embarrassed them. And Howard, best of all, was sober, old enough to know his own mind, and absolutely reliable. He was an economist 40

who had sense enough to attach himself to a corpora- tion that continued to pay his salary during his loan to the government. There was no reason for the engagement or the marriage to fail.

Carol, with great efficiency, nearly at once set 45

about the business of falling in love. Love required only the right conditions, like a geranium. It would wither exposed to bad weather or in dismal surround-ings; indeed, Carol rated the chances of love in a cottage or a furnished room at zero. Given a good 50

climate, enough money, and a pair of good-natured, intelligent (her college lectures had stressed this) people, one had only to sit back and watch it grow. All winter, then, she looked for these right conditions in Paris. When, at first, nothing happened, she blamed 55

it on the weather. She was often convinced she would fall deeply in love with Howard if only it would stop raining. Undaunted, she waited for better times.

Howard had no notion of any of this. His sudden proposal to Carol had been quite out of character— 60

he was uncommonly cautious—and he alternated between a state of numbness and a state of self-congratulation. Before his engagement he had sometimes been lonely, a malaise he put down to overwork, and he was discontented with his bachelor 65

households, for he did not enjoy collecting old pottery or making little casserole dishes. Unless he stumbled on a competent housemaid, nothing ever got done. This in itself would not have spurred him into marriage had he not been seriously unsettled by the 70

visit of one of his sisters, who advised him to marry some nice girl before it was too late. “Soon,” she told him, “you’ll just be a person who fills in at dinner.”

Howard saw the picture at once, and was deeply moved by it. 75

(1953)

Line

Page 80: 2004 AP English Literature and Composition Free …bayareatutoring.com/APlitprompts.pdf · AP® English Literature and Composition 2004 Free-Response Questions Form B ... present,

2003 AP® ENGLISH LITERATURE AND COMPOSITION FREE-RESPONSE QUESTIONS

Copyright © 2003 by College Entrance Examination Board. All rights reserved. Available to AP professionals at apcentral.collegeboard.com and to

students and parents at www.collegeboard.com/apstudents.

4

Question 3

(Suggested time— 40 minutes. This question counts as one-third of the total essay section score.) According to critic Northrop Frye, “Tragic heroes are so much the highest points in their human landscape that they seem the inevitable conductors of the power about them, great trees more likely to be struck by lightning than a clump of grass. Conductors may of course be instruments as well as victims of the divine lightning.” Select a novel or play in which a tragic figure functions as an instrument of the suffering of others. Then write an essay in which you explain how the suffering brought upon others by that figure contributes to the tragic vision of the work as a whole. You may choose a work from the list below or another novel or play of comparable quality. Avoid mere plot summary.

An American Tragedy Anna Karenina Antigone Beloved Crime and Punishment Death of a Salesman Ethan Frome Faust Fences For Whom the Bell Tolls Frankenstein Hedda Gabler King Lear

Light in August Long Day’s Journey into Night Lord Jim Macbeth Medea Moby-Dick Oedipus Rex Phèdre Ragtime Sent for You Yesterday Tess of the D’Urbervilles Things Fall Apart

END OF EXAMINATION